Civ Pro Practice Multiple Choice Questions

Réussis tes devoirs et examens dès maintenant avec Quizwiz!

Imagine that, during a driving trip from pennsylvania to maine, donald takes a wrong turn and, unbeknownst to him, drives into vermont. Before donald discovers his error, his car collides with penelope's. Penelope sues donald in vermont, and donald moves to dismiss the case for lack of personal jurisdiction. Assuming that vermont's long arm statute authorizes jurisdiction over the action, the motion should be A. Denied, because the case arises out of donald's contacts in vermont B. Granted, personal jurisdiction would be unreasonable in this case, even if donald's accident is considered to be a contract with vermont C. Granted, donald did not purposefully avail himself of the privilege of driving in vermont because he drove into vermont by mistake. As a result, he lacked any constitutionally cognizable contacts in vermont D. Granted, not only did donald lack any contacts in vermont, but the exercise of personal jurisdiction would be unreasonable under the circumstances

A. Denied, because the case arises out of donald's contacts in vermont

Perri collided with another vehicle while driving a co-worker, Parra, home from work. Perri sued Drazen (the driver of the other vehicle), alleging that Drazen caused the accident. The jury concludes that Drazen was not negligent, and the court enters a judgment in favor of Drazen. Subsequently, Parra sues Drazen for his own injuries from the accident. Drazen raises the affirmative defense of claim preclusion and moves for summary judgment on these grounds. The court should — A. Deny the motion because Parra was not a party to the first case B. Grant the motion, because Parra had a sufficient relationship with Perri to give rise to non-party preclusion C. Deny the motion, but only if Parra's injuries were more severe than Perri's and Parra had a greater incentive to litigate the case zealously D. Grant the motion because it would be unfair to require Drazen to defend this case again

A. Deny the motion because Parra was not a party to the first case

Fortes sues Berrier for breach of an alleged oral contract to share Berrier's winnings at all his poker tournaments during a given year. He claims that Berrier won $50,000 at the Vegas Texas Hold'em tournament that year but refused to pay half to fortes. Berrier raises several defenses: he argues that he did not make the contract. He also claims that the contract was an illegal contract for the division of gambling winnings and therefore void. Last, he argues that he did not agree to split his winnings for any tournaments if fortes failed to provide the "stake" (funds needed to enter) and that fortes had failed to stake him for the Vegas tournament. Before trial, both parties move for summary judgment on the first two defenses, that the parties did not make the contract, and that the contract is illegal. The judge grants summary judgment to fortes on these two defenses, finding as a matter of law that they did make the contract and that the contract is valid. The case goes to trial, and Fortes loses, because the jury finds that fortes had agreed to provide the stake but failed to do so. Subsequently, Fortes sues Berrier for a later breach of the same contract, claiming that Berrier won $100,000 at the world series of poker, but failed to split the winnings with him. Berrier argues as a defense that they never made the contract and that the contract is illegal. Which of the following statements is correct? A. Berrier should be precluded from asserting either defense, since the judge found against him on those issues in the prior case B. Berrier is not precluded from relitigating either issue, since neither was essential to the judgment in the prior case C. Berrier is not precluded on either issue, since they were not actually decided in the first case D. Berrier is not precluded from litigating either issue, because these issues were not decided by the jury in the prior action

B. Berrier is not precluded from relitigating either issue, since neither was essential to the judgment in the prior case

Pietra has a claim against donnor, a california citizen, that arose out of an injury that pietra suffered while jogging on a rental property in north carolina that donnor owned. Pietra sues donnor in north carolina and asks the north carolina court to attach donnor's property at the outset of the case in an attempt to establish quasi in rem jurisdiction. Donnot's property is valuabe enough to cover pietra's damages if she wins. Assume that donnor purchased the property only to rent it out and has never been to north carolina. (donnor purchased the property from california). donnor has no other contacts with north carolina. donnor moves to dismiss the case for lack of personal jurisdiction. Under these facts, most courts would probably — A. Grant the motion, because donnor has never been to north carolina B. Deny the motion, because this case arises out of donnor's contracts with north carolina C. Deny the motion, because donnor is at home in north carolina and is thus subject to general jurisdiction there D. Grant the motion, because this case does not arise out of donnor's contacts with north carolina

B. Deny the motion, because this case arises out of donnor's contracts with north carolina

Penny believes that demotion, inc. fired her because of her gender, so she brings a federal law gender discrimination claim in the federal court for the northern district of New York. Demotion wins the case at trial, and the court enters judgement in favor of demotion. Penny subsequently files a state law age discrimination claim against demotion in New York state court, alleging that her firing was due to her age. Demotion's answer raises the affirmative defense of claim preclusion, and demotion later moves for summary judgement on that basis and includes a copy of the prior judgment. If the court applies the transactional definition of a claim, the court should - A. Grant the motion B. Deny the motion because age and gender discrimination are different types of discrimination C. Deny the motion because the evidence that penny would need to prove her gender discrimination cause of action is different from the evidence, she would need to prove her age discrimination cause of action D. Deny the motion because claim preclusion does not bar the claim unless the previous lawsuit was litigated in the same court; in this case, the first lawsuit was resolved in federal court, not state court

A. Grant the motion

Rosario sues Demerest in state court, claiming that Demerest fired him based on his age, a violation of the federal age discrimination in employment act. The state court should - A. Hear the suit B. Hear the case only if the parties are not diverse C. Dismiss the case, because it arises under federal law and must be filed in federal court D. Dismiss the case, unless congress has expressly authorized bringing such federal claim in state court

A. Hear the suit

Susan Gordon, after growing up in Pennsylvania, goes to ricks college, in Idaho, planning to get a two year nursing degree and return to practice nursing in Pennsylvania. Two months after starting school, she files suit in a Pennsylvania court federal district court against Dr. Rodriguez, a Pennsylvania citizen who treated her in Pennsylvania for her injury. She claims jurisdiction based on diversity. The court probably — A. Lacks diversity jurisdiction, because Gordon is still domiciled in Pennsylvania B. Lack diversity jurisdiction, because the treatment took place in Pennsylvania C. Has diversity jurisdiction, because she was living in Idaho when she filed the suit D. Would have diversity jurisdiction, if she brought the action in an Idaho federal court

A. Lacks diversity jurisdiction, because Gordon is still domiciled in Pennsylvania

In which one of the following cases is the amount-in-controversy requirement met so as to allow adjudication of all claims in federal court? (Assume the complete diversity requirement is satisfied.) A. Leonard sues Roberts seeking $40,000 for a slander cause of action and also seeks $40,000 for an unrelated breach of contract. B. Leonard sues Roberts seeking $50,000 for losses he suffered because of her breach of a business contract, and joins Lampley as co-defendant seeking $70,000 from her for his losses in the same deal. C. Leonard sues Roberts seeking $50,000 for losses he suffered in a business deal and Cochran joins as co-plaintiff seeking $60,000 from her for his losses in the same deal. D. Leonard sues Roberts seeking $80,000 for injuries suffered in an auto accident, but Cochran joins as a co-plaintiff seeking $50,000 from Roberts for the damages he suffered because of an unrelated breach of contract by her.

A. Leonard sues Roberts seeking $40,000 for a slander cause of action and also seeks $40,000 for an unrelated breach of contract.

Cecilia Doaks failed to pay her income taxes for several years, and the IRS gave her notice by certified mail that it would take custody of and sell her strip mall to cover the taxes she had failed to pay. She failed to redeem the strip mall by paying the delinquent taxes, and, at the tax sale, Della Sands offered the prevailing bid of $180,000 and took title to the strip mall. Ms. Doaks subsequently discovered a federal statute that, arguably, requires in-hand, personal service of process before holding a tax sale. She therefore sued Ms. Sands (both are citizens of Texas) in a federal court in Texas based solely on a Texas quiet title cause of action, and she seeks $180,000 in damages. One of the elements of this state claim requires a plaintiff to prove the defendant has a defective title. Ms. Doaks believes that, because the IRS failed to satisfy the notice requirement in the federal statute, the title it conveyed to Ms. Sands is defective. Ms. Sands responded to this suit by filing a Rule 12(b)(1) motion to dismiss this suit for lack of federal subject matter jurisdiction. (The IRS is not a party.). Should the federal court dismiss this suit? A. No, Ms. Doaks asserts a state claim, but one of its elements depends on a disputed federal issue and leaving that issue to state courts would seriously hinder the operations of the IRS. B. Yes, this is like the Mottley case in that the well-pleaded allegations show that this claim does not arise under federal law. C. No, the jurisdictional amount is sufficient to satisfy diversity jurisdiction. D. Yes, without exception, federal question jurisdiction depends on proof that the plaintiff has asserted a cause of action created by a federal statute.

A. No, Ms. Doaks asserts a state claim, but one of its elements depends on a disputed federal issue and leaving that issue to state courts would seriously hinder the operations of the IRS.

Two citizens of Texas were riding in a car that was struck by an individual who was domiciled in Arizona. The two Texans sued in federal court in Texas asserting state tort claims and seeking damages for their injuries. Plaintiff-1 was driving the car in which he and his friend were injured by the alleged negligence of the Arizonan defendant. He was seriously injured and seeks $400,000 in damages. Plaintiff-2 was a passenger in the car with Plaintiff-1 when it was struck by the defendant. However, he was in the front passenger seat and the defendant's car struck the driver's side nearest Plaintiff-1. Plaintiff-2 suffered minor injuries and joined in Plaintiff-1's suit seeking $10,000 in damages. The Arizonan defendant has moved to dismiss Plaintiff-2's claim for failure to satisfy the amount requirement of the diversity statute. Should the federal judge dismiss Plaintiff-2's claim for failure to satisfy the amount requirement? A. No, Plaintiff-2's claim arose from the same occurrence that gave rise to Plaintiff-1's claim and supplemental jurisdiction supports Plaintiff-2's claim. B. Yes, multiple plaintiffs cannot aggregate their claims to satisfy the jurisdictional amount. C. No, multiple plaintiffs can always aggregate their claims if they arise from the same transaction or occurrence. D. Yes, plaintiffs cannot use supplemental jurisdiction in a diversity case.

A. No, Plaintiff-2's claim arose from the same occurrence that gave rise to Plaintiff-1's claim and supplemental jurisdiction supports Plaintiff-2's claim.

Mary Bird, a citizen of Utah, sued Hilbert Carter, a citizen of Texas, in federal court in Texas. She alleged that his breach of their contract caused her $300,000 in damages. She had him served by leaving the summons and a copy of the complaint with Carter's wife at his home in Alamo Heights in accordance with Rule 4. On the day of service, however, Mr. Carter's wife ran away with another man and failed to notify him of the process papers she had received. As a result, he did not file a timely answer, and a default judgment was entered in Ms. Bird's favor. Several months later, Mr. Carter's wife finally delivered the process to him. Mr. Carter thereafter moved in federal court to set aside the default judgment because, he contends, the court violated his due process rights by not providing him with actual notice of the suit against him. He argues that, in the absence of actual notice, the default judgment was entered in violation of due process. Must the court vacate this judgment as being in violation of due process? A. No, a defendant has a due process right to service by a means reasonably calculated to provide actual notice, but actual notice is not a constitutional requirement. B. Yes, Mr. Carter was denied actual notice because of circumstances beyond his control, and due process requires more of the judicial system. C. No, although a defendant has a due process right to actual notice, the failure of notice in this case was the fault of Mr. Carter. D. Yes, the actions of Mrs. Carter in denying her husband notice denied him due process.

A. No, a defendant has a due process right to service by a means reasonably calculated to provide actual notice, but actual notice is not a constitutional requirement.

In 2010, while living in Indiana, Patti Jones purchased a $100,000 life insurance policy from the Good-Bye Life Insurance Co. (GBL), which is incorporated and has its principal place of business in Nevada. In 2012, Ms. Jones moved to Texas to live with her mother because she was suffering from an illness. Until 2016, she continued to make payments on the insurance policy from Texas, and these checks were received in Nevada and cashed by GBL. In 2016, however, GBL cancelled the insurance policy because Ms. Jones had failed to make one month's premium payment. After further negotiation, GBL sent a renewal offer for a term life policy at a face value of $100,000 to Ms. Jones in Texas. She accepted and was thereafter current in her premium payments. Ms. Jones died in 2018, and her mother, Mrs. Joyce Jones (the beneficiary under the policy), requested payment from GBL. GBL refused to pay her claim, contending that Ms. Jones committed suicide, which violates a contractual limitation. Mrs. Jones contends that her daughter died of cancer. In 2018, Mrs. Jones sued the company for breach of contract in a state court in Texas seeking $100,000. GBL has filed the appropriate papers seeking to dismiss this suit. Attached to its motion, GBL filed affidavits showing that Ms. Jones was the only insured it had in Texas, that it has no office or employee in Texas, and that it has never advertised for business in the state. Should the Texas court dismiss this suit? A. No, because GBL established this 2016 contractual relationship by sending a renewal policy to Ms. Jones after she moved to Texas. B. No, because the Texas court has diversity jurisdiction over this case. C. Yes, because the initial contractual relationship with Texas came about because of the unilateral action of Ms. Jones. D. Yes, because the Texas court has removal jurisdiction.

A. No, because GBL established this 2016 contractual relationship by sending a renewal policy to Ms. Jones after she moved to Texas.

Harry Bosch is a private detective in California, and he was hired by a rich Texan, Hiram Soles, to find Mr. Soles' wife. Mr. Soles resides in Dallas in a very expensive neighborhood. Mr. Bosch traveled through Texas trying to find Mrs. Soles. He finally found her in Corpus Christi, and she was living with another man. When Mr. Bosch confronted her, and demanded that she return to her husband, she showed him the scars that had been inflicted on her by her husband. Mrs. Soles pleaded with Mr. Bosch not to give her location to her husband. After returning to California, Mr. Bosch called Mr. Soles and told him he could not perform their contract. Mr. Soles assumed correctly that Mr. Bosch knew where his wife was living and was enraged by Mr. Bosch's refusal to disclose that information. Mr. Soles then brought a diversity action in federal court in Dallas seeking specific performance of his contract with Mr. Bosch. Specific performance is an equitable remedy that consists largely of a permanent injunction. Mr. Bosch appeared in this suit in federal court and asserted a counterclaim against Mr. Soles seeking damages equal to the expenses he incurred in seeking to find Mrs. Soles. Mr. Bosch demanded a jury trial for his counterclaim because he had concluded that a jury was not likely to favor Mr. Soles after evidence was introduced of his abuse. Mr. Soles moved to try all issues common to his contract claim and Mr. Bosch's contract counterclaim before the judge and moved to quash Mr. Bosch's demand for a jury trial because this case was predominantly equitable. Should the federal court grant Mr. Soles' motion to quash? A. No, because all of the issues common to the claim and counterclaim must be submitted to the jury. B. Yes, because the district court can conclude that this litigation is predominantly equitable. C. No, because the jury should always have the opportunity to judge someone as despicable as this plaintiff. D. Yes, because a plaintiff's claim must always be determined before a counterclaim.

A. No, because all of the issues common to the claim and counterclaim must be submitted to the jury.

Sonja Henie was an ice skater who earned a living appearing at ice shows. At one ice show in Houston, she was injured because of cracks in the ice. Houston Ice Shows, Inc., is a Texas corporation, and Ms. Henie is a citizen of New York. Ms. Henie brought suit seeking $400,000 in damages from Houston Ice Shows. In this suit in federal court based on diversity, Ms. Henie was joined as a plaintiff by her mother, who is also a citizen of New York. Her mother had suffered emotional distress because of the injury to her daughter and had provided nursing care for her as well. The mother, Mrs. Henie, sought $50,000 in damages. Houston Ice Shows filed a motion to dismiss the mother's claim for lack of subject matter jurisdiction. Should the federal court dismiss the mother's claim for this reason? A. No, because the court has supplemental jurisdiction over this added claim, which arises from the same nucleus of operative fact as the daughter's claim. B. Yes, because the mother's claim does not satisfy the jurisdictional amount. C. No, because the mother's claim does satisfy the jurisdictional amount. D. Yes, because federal subject matter jurisdiction only extends to claims for more than $75,000.

A. No, because the court has supplemental jurisdiction over this added claim, which arises from the same nucleus of operative fact as the daughter's claim.

Ten female employees of Toys for Tots, Inc. (Toys) have sued that company in a federal court in Texas asserting a federal cause of action for employment discrimination because of sex. The company is incorporated in Delaware and has its executive offices in Houston, Texas. Four of the plaintiffs are domiciled in states other than Texas, but six are domiciled in Texas. Each of the ten employees complains about a company policy applied in the different stores where they work that discriminates against women in regard to promotions. Each contends that she was denied a promotion, which was then given to a less qualified male employee. Toys complains that each of the ten plaintiffs were denied a promotion at a different Toys' store, on a different date, and in regard to promotion to different jobs. It therefore moves to sever the claims into ten suits. Should the federal court grant this motion to sever these claims? A. No, because the plaintiffs complain about the same discriminatory company policy arising from a series of related transactions giving rise to a common issue. B. No, because the joinder of parties is not restricted by the Federal Rules of Civil Procedure. C. Yes, because the litigation of ten claims in one suit poses an impossible burden on the court. D. Yes, because these ten claims arose from transactions that occurred at different times.

A. No, because the plaintiffs complain about the same discriminatory company policy arising from a series of related transactions giving rise to a common issue.

In 1939, the Hardaway River Authority (the Authority), an Illinois agency, began building a dam on the Hardaway River to protect the city of Hamburg, Illinois, from flooding by the river. Hamburg had for many years suffered from floods that occurred largely because a heavy rain could easily cause the river to flow over its banks into the city. The Authority planned to use the dam to protect the city by permanently lowering the water level between the dam and Hamburg by 10 feet. Fifty families own property upstream from Hamburg and downstream from the proposed dam. They live on the banks of the river in an area known as the Hardaway River Subdivision (HRS). All 50 homeowners in the HRS have boats and fishing piers that would become useless if the water level were reduced by 10 feet. Roland Snow, a homeowner in HRS, sued the Authority in an Illinois state court. He asked that court to declare the planned dam an unconstitutional deprivation of the property rights of the HRS homeowners and asked the court to permanently enjoin its construction. At this time, no Illinois procedural rule existed to authorize a class action, but Mr. Snow alleged that he was bringing this action as a representative of all homeowners in the HRS, and he provided notice to each homeowner. As Mr. Snow's case proceeded to judgment, some of the homeowners joined Mr. Snow in the suit, others began protesting the planned dam outside the courthouse, and many others provided money to pay Mr. Snow's lawyers. In 1941, the trial court ruled that Mr. Snow had no right to prevent construction of the dam. Exhausted and out of money, Mr. Snow did not appeal. However, Haley Frost, Mr. Snow's next-door neighbor brought the same claim in an Illinois state court seeking to prevent construction of the dam. The Authority moved to dismiss her suit, contending that she was bound by the judgment in Mr. Snow's suit as a member of the class he represented. Ms. Frost responded to this motion by contending that holding her bound by the judgment in Snow's case would deprive her of due process. If denied the ability to sue, would Ms. Frost be deprived of due process? A. No, because this was a class action in that Ms. Frost's interests were fully represented by Mr. Snow in his suit, and that representation satisfied due process. B. Yes, she cannot be bound by this judgment because she chose not to join as a named plaintiff. C. No, she was bound because of virtual representation even though she was not in privity with Mr. Snow. D. Yes, she cannot be bound as an unnamed member of the class because her interests conflicted with those represented by Mr. Snow.

A. No, because this was a class action in that Ms. Frost's interests were fully represented by Mr. Snow in his suit, and that representation satisfied due process.

Jack "the Troll" Teeter operates a website from his home in Oklahoma and authors a podcast that one can access on the Internet. His podcast features his weekly monologues concerning politics and has an average audience of over 100,000 persons. Texas residents make up more than half of his regular audience. Over the last few months, he has criticized Texas state officials for their handling of mask mandates in schools. During his last rant about these Texas policies, he called Texas Governor Abbott a "maniac." He also described the Governor's objections to public school mask mandates as "obviously designed to do the maximum damage to school children." Mr. Teeter ended his rant by calling on Texas voters to run "that monster out of the state" and stated that Governor Abbott was a direct descendant of Satan himself. The Austin American Statesman carried a story about Mr. Teeter's contentions and noted that his is one of the most popular podcasts in Austin. Governor Abbott filed suit in an Austin federal court based on diversity jurisdiction asserting a libel claim against Mr. Teeter seeking $1.5 million dollars. (He did not include the Austin American Statesman.) Mr. Teeter filed a motion to dismiss this suit for lack of personal jurisdiction. He claims that just because Governor Abbottis whining about his hurt feelings does not create specific jurisdiction in Texas. Mr. Teeter also added to his motion that he has never set foot in Texas, has never done business in the state, and has no property there. He then ended his motion with a drawing of what appears to be a rude hand gesture. Should the federal court dismiss this suit? A. No, the federal district court has diversity and personal jurisdiction. B. Yes, Mr. Teeter cannot have a purposeful contact with Texas if he has never set foot in the state. C. No, Mr. Teeter is subject to general jurisdiction in Texas. D. Yes, Mr. Teeter cannot be within the specific jurisdiction of a court in Texas because he was not engaged in business in the state.

A. No, the federal district court has diversity and personal jurisdiction.

Abigail Almarez, a citizen of Texas, sued Henry Caldicott, a citizen of New Jersey, in a Texas state court. Ms. Almarez and Mr. Caldicott had lived together in his apartment in New Jersey until she broke up with him and moved back to Texas. He retaliated against her for leaving him by posting on the Internet a photo taken of her while she was nude. She had allowed this photograph only after getting his assurance that he would never make it public. Because of his actions, she asserted a cause of action for invasion of privacy in a Texas state court seeking $1,000,000 in damages. Mr. Caldicott has never been in Texas, and he hired a Texas lawyer only to challenge the personal jurisdiction of the Texas court. The court in Texas held that Mr. Caldicott was subject to jurisdiction in Texas. Mr. Caldicott did not appeal and refused to participate in any further litigation in the Texas court. The Texas state court entered a default judgment for $300,000, and Ms. Alvarez took that Texas state court judgment to a New Jersey state court in an enforcement action, which was necessary before New Jersey would make the judgment one of its own. He appeared before the New Jersey court and moved to dismiss her suit solely because it was based on a Texas judgment that was entered without personal jurisdiction. Should the New Jersey court grant Mr. Caldicott's motion to dismiss? A. No, the issue of the Texas court's personal jurisdiction has been decided, and Mr. Caldicott is precluded from challenging that issue again. B. No, Mr. Caldicott's default on the merits prevents him from making a jurisdictional challenge. C. Yes, the Texas court clearly erred in assuming personal jurisdiction over Mr. Caldicott, and he can therefore prevent enforcement of that judgment. D. Yes, the suits in Texas and New Jersey were based on different claims, and for that reason the New Jersey litigation is not precluded.

A. No, the issue of the Texas court's personal jurisdiction has been decided, and Mr. Caldicott is precluded from challenging that issue again.

Abigail Almarez, a citizen of Texas, sued Henry Caldicott, a citizen of New Jersey, in a federal court in Texas. Ms. Almarez and Mr. Caldicott lived together for several years in his apartment in Jersey City, New Jersey, in a very expensive apartment building that Mr. Caldicott owns. Earlier this year, however, Ms. Almarez broke up with Mr. Caldicott and moved back to live with her mother in San Antonio. After she broke up with him, he retaliated by posting on the Internet a photo taken in his apartment of Ms. Almarez while she was nude. She posed for this photograph only after getting his assurance that he would never make it public. Because of his actions, she asserted a cause of action for invasion of privacy in a Texas state court seeking $1,000,000 in damages. Mr. Caldicott has never been in Texas, and he hired a Texas lawyer only to challenge the personal jurisdiction of the Texas court. After the parties were allowed discovery concerning personal jurisdiction, the Texas court held a hearing at which both sides appeared. After a three-day hearing, the Texas state trial court decided that it did, in fact, have personal jurisdiction over Mr. Caldicott because he placed the photo on the Internet. He thus made it accessible by friends and employers of Ms. Almarez in Texas. [This ruling rests on a controversial interpretation of the minimal-contacts requirement for personal jurisdiction and, had Mr. Caldicott appealed the ruling, it might have been overturned by a Texas appellate court.] Mr. Caldicott did not, however, appeal and was so horrified by this interpretation of the law that he refused to participate in any further litigation in a Texas court. The Texas state court thereafter determined that Ms. Almarez's damages should be only $300,000 and entered judgment for that amount. She then took that Texas state court final judgment to a New Jersey state court in an enforcement action, which was necessary before New Jersey would make the judgment one of its own. Once New Jersey accepted the Texas judgment, she could execute her judgment against Mr. Caldicott's New Jersey assets. He appeared before the New Jersey court and moved to dismiss her suit solely because it was based on a Texas judgment that was entered without personal jurisdiction. Ms. Almarez opposes his motion to dismiss contending that his personal-jurisdiction issue is precluded. Should the New Jersey court grant Mr. Caldicott's motion to dismiss? A. No, the issue of the Texas court's personal jurisdiction has been decided, and Mr. Caldicott is precluded from challenging that issue again. B. No, Mr. Caldicott's default on the merits prevents him from making any jurisdictional challenge. C. Yes, the Texas court clearly erred in assuming personal jurisdiction over Mr. Caldicott, and he can therefore prevent enforcement of that judgment. D.Yes, the suits in Texas and New Jersey were based on different claims, and for that reason claim preclusion bars the New Jersey litigation

A. No, the issue of the Texas court's personal jurisdiction has been decided, and Mr. Caldicott is precluded from challenging that issue again.

Which one of the following statements best describes the Supreme Court's holding in the Iqbal decision? A. The plaintiff must plead facts that, when taken as true, establish the liability of the defendant. B. The plaintiff may plead facts that, when taken as true, support the conclusion that the defendant's actions could equally be deemed unlawful or lawful. C. The plaintiff must state the elements of her cause of action and allege that they will all be satisfied by facts gained through formal discovery. D. The plaintiff must allege conclusions that, if taken as true, establish the liability of the defendant

A. The plaintiff must plead facts that, when taken as true, establish the liability of the defendant.

Bob Amherst Metals, Inc. (BAM), is incorporated in Texas with its only place of business in Beaumont, Texas, and its sole business is to produce home water heaters. BAM sells all its water heaters to Acme Distributors, Inc., (Acme), which is incorporated in Texas with its only place of business in Dallas. BAM owns a controlling share of Acme's stock and requires Acme to buy heaters only from BAM and to sell all 1250 units it buys from BAM in Texas, Arkansas, and Oklahoma. In the summer of 2020, Amir Al-Abed purchased one of BAM's water heaters from a retail store in Little Rock, Arkansas. After installing the water heater in his Little Rock home, it exploded and destroyed his home and caused him serious physical injuries. Experts hired by Mr. Al-Abed determined that defective metal tubing caused the explosion. Mr. Al-Abed has now filed suit in federal court in Arkansas seeking $5 million in damages and has named both corporations as defendants. BAM have moved to dismiss this suit against it for lack of personal jurisdiction. Should the federal court dismiss this suit? A. No, unlike the Asahi decision, BAM is not a foreign corporation, it had greater control over the stream of commerce, and the forum state has a strong interest in protecting its own citizens from defective products. B. Yes, like the Asahi decision, BAM would be greatly inconvenienced by the obligation to litigate in Arkansas and had no control over the ultimate sale of its heaters. C. No, the federal court has diversity jurisdiction and does not therefore need personal jurisdiction. D. Yes, BAM sold all its heaters in Texas to Acme and therefore had no purposeful contact with Arkansas.

A. No, unlike the Asahi decision, BAM is not a foreign corporation, it had greater control over the stream of commerce, and the forum state has a strong interest in protecting its own citizens from defective products.

Pietra has a claim against Donner, a California citizen, that arose out of a car accident that occurred in California. Pietra is a citizen of North Carolina and asks a North Carolina court to attach a money market account that Donnor owns in the state and that he opened while visiting the state. Pietra then seeks to litigate the car accident case in North Carolina, resigning herself to recover only the funds in donor's North Carolina money market account. Assume Donnor's North Carolina account is completely unrelated to Pietra's underlying claim against Donnor and assume that Donnor does not otherwise have contacts with North Carolina sufficient to subject him to in personam jurisdiction in the state. Under these facts, most courts would probably - A. Not exercise quasi in rem jurisdiction because Donnor lacks sufficient contacts with the state B. Not exercise quasi in rem jurisdiction because this basis for jurisdiction was eliminated by the supreme court's Shaffer opinion C. Exercise quasi in rem jurisdiction as long as the state court attached Donnor's account at an inappropriate time in the case D. Exercise in personam jurisdiction because the supreme court has eliminated the distinction between quasi in rem and in personam jurisdiction

A. Not exercise quasi in rem jurisdiction because Donnor lacks sufficient contacts with the state

Mr. Dupont is a citizen of France, and he married Mrs. Dupont, a U.S. citizen, when both were domiciled in Texas while attending college in San Antonio. After finishing their degrees, they moved to France and took jobs there. They then brought a suit in federal court in Texas against Homer Jarman, a citizen of Texas. They sued Mr. Jarman on the basis of a state-law claim for invasion of privacy and seek $500,000 each as damages. They had learned that Mr. Jarman rented them an apartment in San Antonio in which he had planted hidden video cameras. They also learned that Mr. Jarman had downloaded the videos of their intimate relations to his computer and had shared these with some of his friends. Mr. Jarman has filed a motion in the federal court to dismiss Mrs. Dupont's claim in this suit. Which one of the following describes the correct ruling on that motion? A. The court lacks jurisdiction because Mrs. Dupont is not a citizen of a state or of a foreign country. B. The court lacks jurisdiction because Mrs. Dupont remains a citizen of Texas. C. The court has jurisdiction because Mrs. Dupont is a citizen of Texas. D. The court has jurisdiction because Mrs. Dupont's move to France makes her a citizen of that country.

A. The court lacks jurisdiction because Mrs. Dupont is not a citizen of a state or of a foreign country.

Assume that Mr. and Mrs. Mas were both domiciled in Louisiana while in school, but after finishing their degrees they move to France, planning to live there indefinitely. After arriving in France, they bring suit on a state law tort claim against perry in federal court in Louisiana. A. The court lacks jurisdiction, because Mrs. mas is not a citizen of a state or a foreign citizen B. The court lacks jurisdiction, because Mrs. mas remains a citizen of Louisiana C. The court has jurisdiction, since neither Mr. Mas nor Mrs. mas is domiciled in Louisiana D. The court has jurisdiction, because Mr. and Mrs. Mas have become foreign citizens. Thus, the case is between a state citizen, Mr. Perry, and citizens of a foreign state

A. The court lacks jurisdiction, because Mrs. mas is not a citizen of a state or a foreign citizen

In March of 2019, Bill Cleaver, a citizen of Texas, was injured when he suffered three superficial cuts because the power drill he was using threw off sharp metal shards. He was using this drill in Austin, Texas, while working for a construction company. Mr. Cleaver was taken to the hospital where doctors put antibiotic lotion on the three cuts and provided him with Motrin, an anti-inflammatory pain reducer. He missed one day of work and was assigned to office work for the remainder of the week. He did not require any further medical treatment. Three months later, Mr. Cleaver brought a tort suit for negligence and defective design against Adarand Tools, Inc. seeking $100,000 in damages. Adarand manufactured the drill which caused his injuries. Mr. Cleaver filed this suit in the United States District Court for the Western District of Texas, Austin Division. The defendant, Adarand, is incorporated in Delaware, and has its headquarters in California. Adarand sold the power drill to this Texas construction company after the company responded to Adarand's advertising in Texas. It has now filed a preanswer motion to dismiss this suit. Which of the following is the correct response to Adarand's motion to dismiss? A. The federal court should dismiss because it lacks subject matter jurisdiction. B. The federal court should dismiss because it lacks personal jurisdiction. C. The federal court should not dismiss because it has subject matter jurisdiction over this diversity case. D. The federal court should not dismiss because Adarand has waived its objections to subject matter and personal jurisdiction.

A. The federal court should dismiss because it lacks subject matter jurisdiction.

[In the litigation described above, other problems occurred in Sleep Number federal lawsuit against Saginaw.] Saginaw filed the third-party complaint against Hacksaw and Ginger Paint, which first prompted an answer by Ginger Paint, which included both a counterclaim against Saginaw and a cross-claim against Hacksaw. After receiving a copy of Ginger Paint's answer, Hacksaw answered the third-party complaint and included its own counterclaim against Saginaw and two claims against Ginger Paint. Hacksaw's first claim against Ginger Paint contended that the latter's construction delays caused $1 million damages to Hacksaw. Hacksaw's second claim arose from a previous contract Ginger Paint had with Hacksaw when the two of them were constructing a building in Kansas. In that contract, Hacksaw says Ginger Paint was supposed to pay it $400,000 in fees, which it never paid. Ginger Paint has filed a motion to dismiss both of Hacksaw's claims against it. Can the federal court adjudicate Hacksaw's two claims? A. Yes and No, the first claim is a compulsory counterclaim supported by supplemental jurisdiction, but the second claim arises from a different transaction and is not supported by supplemental jurisdiction. B. Yes, once Hacksaw asserted a proper compulsory counterclaim, Federal Rule 18 allows it to join any related or unrelated claims it has against Ginger Paint. C. No, both Hacksaw and Ginger Paint are Missouri corporations, and neither claim asserts a federal cause of action. D. Yes, claims between these third-party defendants are supported by the jurisdiction established when Sleep Number filed its original complaint.

A. Yes and No, the first claim is a compulsory counterclaim supported by supplemental jurisdiction, but the second claim arises from a different transaction and is not supported by supplemental jurisdiction.

Paul Peterson, a citizen of Oregon, sued David Dalton, a citizen of Minnesota, in an Oregon state court alleging that Dalton had caused Peterson damages because of a breach of contract. Peterson sought $400,000 in damages, and Dalton answered based on his contention that he had performed the contract. Although this case was removable as filed, Dalton did not remove it. However, 90 days later, Peterson amended his complaint to add a federal cause of action for infringement of his federal trademark that Dalton had used without permission. On the day he received this amended complaint, Dalton removed the case to federal court. Peterson has now filed a timely motion to remand. Should the federal court remand this case? A. Yes, because the 30-day removal period began running when this otherwise removable diversity case was originally filed in state court, and Dalton failed to remove in time. B. No, because a defendant has 90 days within which to remove a case to federal court. C. Yes, because neither diversity nor federal-question jurisdiction exists in this case. D. No, because a federal court has discretion to assume jurisdiction over any case.

A. Yes, because the 30-day removal period began running when this otherwise removable diversity case was originally filed in state court, and Dalton failed to remove in time.

George Kemp was injured in an accident while a passenger on the Pacific Coast Railroad (PCR), which is incorporated in Delaware and has its executive offices in Oregon. The accident occurred in the State of Washington, but he sued PCR in federal court in his home state of California for $600,000 in damages. PCR operates railroad lines in 28 States and has 2,061 miles of railroad track in California (about 6% of its total track mileage of 32,500), employs some 2,100 workers there (less than 5% of its total work force of 43,000), generates less than 10% of its total revenue in the State, and maintains only one of its 24 facilities in California (4%). Mr. Kemp concedes that the court does not have specific jurisdiction but argues that PCR is clearly at home in California because it does continuous and systematic business there. As its first response to the complaint, PCR filed an answer in which it asserted a defense based on its contention that the California federal court lacked personal jurisdiction. Should the federal court in California dismiss this suit for lack of personal jurisdiction? A. Yes, general jurisdiction does not support this suit in California. B. Yes, if PCR had been doing more business in California, it would have been subject to general jurisdiction in that state. C. No, the railroad is "at home" in California because it does a substantial amount of business in that state. D. No, the railroad has waived its objection to personal jurisdiction by failing to file a preanswer motion.

A. Yes, general jurisdiction does not support this suit in California.

After the tug "J.M. Taylor" sank under mysterious circumstances, the tug owners hired Mr. Fortenbaugh, as their lawyer and asked him to prepare for the anticipated lawsuits. Mr. Fortenbaugh interviewed the surviving crewmen and took their signed statements and after an extensive search discovered that a sea captain witnessed the sinking while sitting on his own boat. However, the sea captain believed that the tug was negligently maintained and expressed his contempt for the owners who caused the death of the crewmen because they had saved money on maintenance. Mr. Fortenbaugh took the sea captain's signed statement including his contentions that the tug was badly maintained. Fortenbaugh alerted the tug owners that he will not call the sea captain at trial because he would be a damaging witness. Subsequently, the plaintiff's lawyer sent interrogatories to the tug owners demanding that they identify any eyewitnesses to the sinking of the tug. In response, Mr. Fortenbaugh refused to disclose the sea captain's identity contending that such information results from attorney work product. Should the judge order the tug owners to disclose any eyewitnesses? A. Yes, the fact that the sea captain was an eyewitness cannot be hidden as work product. B. No, Fortenbaugh discovered the sea captain by hard work and should not have to give up this information. C. Yes, the plaintiff may not be able to obtain this information and that fact overrides the work product immunity that would otherwise apply. D. No, Fortenbaugh took the captain's statement and thereby made his identity protected as attorney work product.

A. Yes, the fact that the sea captain was an eyewitness cannot be hidden as work product.

Bill Carr, a citizen of Texas, was physically injured by two Houston police officers, also citizens of Texas. After his recovery (and release from jail), Mr. Carr hired a lawyer to sue the two officers. That lawyer drafted a complaint in which Mr. Carr sought damages based on a Texas tort cause of action—assault and battery. In addition, the lawyer advised Mr. Carr that he had an additional claim under 42 U.S.C. § 1983, a federal statute that creates a cause of action against state or local officials for violation of constitutional rights. Mr. Carr decided to bring this suit in Texas state court, and he asserted only the tort cause of action. Twenty-two days after they were served with process in the Texas state court suit, the two police officers removed the case to federal court. In their Notice of Removal, they contended that this case was essentially a federal constitutional case and therefore could be removed on federal question jurisdiction. They alleged that Mr. Carr possesses a federal cause of action but is trying to avoid federal court and trying thereby to prevent them from removing the case to federal court by failing to assert it. Two months after removal, Mr. Carr filed a motion to remand the case to state court. He contends that the federal court does not have federal question or diversity jurisdiction over this case and therefore must remand it state court. Should the federal judge remand this case to state court? A. Yes, the federal court lacks diversity and federal question jurisdiction. B. No, the plaintiff has waived his right to seek a remand for lack of subject matter jurisdiction. C. Yes, even though the federal court has federal question jurisdiction, the judge should use her discretion to dismiss this suit because it is based on state law. D. No, the defendant can remove this case based on the Grable/Gunn exception to the creation test.

A. Yes, the federal court lacks diversity and federal question jurisdiction.

Bedelia Ahmad is the lead lawyer for the Union Southern Northern Railroad (USNR), which is incorporated and has its principal place of business in Oregon. Earlier this year, one of USNR's freight trains containing 34 cars and listed as # 212 was halted on the railroad's right-of-way adjacent to a residential neighborhood in Seattle, Washington. The train was stopped while railroad workers repaired a loose coupling. Once that repair was completed, the workers got back on the train and it started moving. Under federal law, the engineer of a freight train must sound its horn before the train moves in order to warn railroad workers that they should brace themselves for movement. On this occasion, # 212 lurched forward causing one of the workers to fall off the train between two of the cars. The worker suffered grave injuries. Shortly after this injury and before the suit was brought, Ms. Ahmad took signed statements from the 12 railroad employees, other than the injured employee, who were working on # 212. These workers were uncertain about whether the engineer sounded the horn before he started to move the train, and Ms. Ahmad was sufficiently concerned to begin searching for possible eyewitnesses who resided in homes adjacent to the right-of-way. After discussing the accident with over 20 residents of that neighborhood, she met Mrs. Grace Ogletree. Mrs. Ogletree stated flatly that she was sitting on her back porch having tea and watching the workers do their repairs on # 212. She told Ms. Ahmad that she was surprised when the train lurched into movement without first sounding its horn. She contends that this action surprised the worker whom she saw fall from the train. Ms. Ahmad thanked Mrs. Ogletree and decided not to take her written statement. In discussing this conversation with high officials of USNR, she advised them that Mrs. Ogletree's eyewitness testimony would be damaging to the railroad's defense. The injured worker, George Planck, has now brought suit in federal court based on a federal cause of action allowing railroad workers to sue a railroad when injured. After USNR was served with process, the Mr. Planck's attorney sent a set of interrogatories to USNR. The third one asked whether USNR knew of any eyewitness to the accident other than the 12 workers on the train. USNR refused to answer that interrogatory and contends that such information is protected as work product. The railroad argues that Ms. Ahmad billed it over $15,000 for the time she spent interviewing people living in the neighborhood. Should the federal judge order USNR to give up Mrs. Ogletree's name? A. Yes, the name of an eyewitness known to a party is not that party's work product. B. Yes, the injured worker has a right to the name of an eyewitness whenever he can show a substantial need and an inability to otherwise acquire that information. C. No, the injured worker's attorney seeks to feast off the work of Ms. Ahmad, and that work produced the information he seeks. D. No, the injured worker's attorney cannot demand this information if he can acquire it through his own investigation

A. Yes, the name of an eyewitness known to a party is not that party's work product

Betty Stokes, a citizen of Texas, was working as an independent contractor for City Public Services, the San Antonio provider of electric services when she was injured by a machine that lifts workers to repair electrical lines. She had been called in as an expert on the electrical lines that were in ill repair. While doing this work, the lifting machine abruptly dropped her to the ground and injured her back. Ms. Stokes learned that Big Machines, Inc., a Delaware corporation with its executive offices in Santa Fe, New Mexico, had constructed the lifter and owned the one that had injured her. Ms. Stokes filed suit in federal court in San Antonio against Big Machines seeking damages of more than $300,000. After she filed suit and served Big Machines, that company impleaded Star Rental, Inc., a corporation incorporated in Delaware that also had an office in Santa Fe, contending that Star Rental was the actual owner of the lifting machine and was liable to Big Machines for any amount it was liable to Ms. Stokes. Ms. Stokes thereafter amended her complaint to assert a claim for her injuries against Star Rental seeking her damages. Star admitted in its answer to her amended complaint that it was a citizen of Delaware and of New Mexico. After she dismissed her claim against Big Machines, Star Rental moved to dismiss her suit claiming for the first time that its executive headquarters, and therefore its principal place of business, was in El Paso, Texas. The federal district judge denied Star Rental's motion to dismiss holding that it was estopped from asserting a ground for dismissal that contradicted its own pleadings. Did the judge err in failing to dismiss Ms. Stokes' claim against Star Rental? A. Yes, this claim was not supported by diversity jurisdiction and cannot be supported by supplemental jurisdiction because of § 1367(b). B. No, Star Rental has waived its objection to subject matter jurisdiction by pleading that its principal place of business was in New Mexico. C. Yes, the judge should have dismissed her claim against Star Rental because it was impleaded by Big Machines without subject matter jurisdiction. D. No, jurisdiction over Star Rental is determined solely by asking whether Big Machines' impleader was supported by subject matter jurisdiction.

A. Yes, this claim was not supported by diversity jurisdiction and cannot be supported by supplemental jurisdiction because of § 1367(b).

In October of 2020, Halle Berry, a citizen of California, had agreed in California to make a movie for Texas Movie Studios, Inc. (TMS), a Texas corporation with its principal place of business in Texas. However, after reviewing the 46-page contract that she had signed, Ms. Berry noticed that her salary would come solely from a percentage of theater sales. That was a problem because TMS reserved the right to release the movie on Netflix instead of in theatres. Because that would deny her any compensation, she refused to make the movie. In January 2021, Ms. Berry was flying from California to meet with friends in Florida. While sitting in her first-class seat, she was served by a process server while the United Airlines plane was flying over the panhandle of Texas. The process server handed her a summons and a copy of the complaint indicating that TMS had filed suit against her in a Texas state court for $4 million because of her breach of contract. She has now filed a motion to dismiss this suit for lack of personal jurisdiction. Assuming Ms. Berry had no other contacts with Texas, should this suit be dismissed for lack of personal jurisdiction? A. Yes, unlike Burnham, she has not voluntarily entered or enjoyed any benefits from activities in Texas. B. No, just like Burnham, she was served after voluntarily entering the state toenjoy the benefits of Texas. C. Yes, in the absence of any purposeful contacts, personal service on a defendant in Texas cannot support jurisdiction. D. No, Ms. Berry's purposeful contact came from doing business with acorporation she knew to have been incorporated in Texas.

A. Yes, unlike Burnham, she has not voluntarily entered or enjoyed any benefits from activities in Texas.

Phillip sued dank corp, alleging that danko corp constructed a facility that was located partially on phillip's land. The dispute turned on the precise boundaries on phillip's property. The court granted summary judgment in danko's favor concluding that the facility was built entirely on danko's property. Phillip subsequently sold his land to bob, and bob now sues danko corp, alleging that the same facility that was at issue in phillip's case was built on bob's (formerly phillips) property. Danko raises the affirmative defense of claim preclusion and moves for summary judgment on those grounds, the court should - A. Deny the motion because bob was not a party to the first case B. Grant the motion, because all of the elements of claim preclusion are satisfied C. Deny the motion because there was never a final judgment on the merits in phillip's case D. None of the above

B. Grant the motion, because all of the elements of claim preclusion are satisfied

You represent Corey in his bar fight case and would like to file the case in federal court based on diversity jurisdiction. Assume that Corey is a citizen of Massachusetts, based on his domicile there. Barristers in incorporated in Delaware. It owns and operates eight bars in Massachusetts. It also owns and operates one in Rhode Island, two in Connecticut, and one in Maine. Its corporate headquarters is in Rhode Island. A federal court would - A. Have diversity jurisdiction even if Barrister's principal place of business is in Massachusetts, because Corey is a citizen of Massachusetts and barristers is a citizen of Delaware B. Have diversity jurisdiction because Corey is a citizen of Massachusetts and barristers is a citizen of Rhode Island and Delaware C. Lack diversity jurisdiction because both Corey and barristers are citizens of Massachusetts D. Lack diversity jurisdiction, if the suit is brought in Massachusetts, since Corey is domiciled there

B. Have diversity jurisdiction because Corey is a citizen of Massachusetts and barristers is a citizen of Rhode Island and Delaware

[Assume all claims satisfy the jurisdictional amount.] Saginaw Construction, incorporated in Delaware with its principal place of business in Kansas, entered a general contract to build an office building for the Sleep Number Corporation. Sleep Number is incorporated and has its principal place of business in Missouri, and wanted this office building constructed in that state as well. In its contract with Saginaw, Sleep Number required Saginaw to pay any losses caused by subcontractors. Two of Saginaw's subcontractors were Hacksaw Builders and Ginger Paint and Plaster. Both of these corporations were incorporated and have their principal place of business in Missouri. The office building's construction was severely delayed, and Sleep Number filed suit in a federal court in Missouri against Saginaw seeking $4 million in damages for Saginaw's failure to complete the construction in a timely manner. Saginaw filed a third-party complaint in this suit against Hacksaw and Ginger Paint contending that these subcontractors were liable to it for all or part of what it might be liable to Sleep Number. Ginger Paint responded to this third-party complaint with an answer containing a counterclaim for breach of contract damages against Saginaw and a cross-claim seeking breach of contract damages against Hacksaw. You represent Hacksaw, and its officials complain to you that Ginger Paint caused the delays in construction, which led to Hacksaw's loss of over $1 million in contractual payments. The Hacksaw officials have argued that they would prefer to bring this claim in a state court in Missouri. Can Hacksaw bring its claim in state court instead of in this federal suit? A. Yes, Hacksaw's claim cannot be brought in this federal suit because no diversity exists between Hacksaw and Ginger Paint. B. No, Hacksaw's claim is a compulsory counterclaim and will be supported by supplemental jurisdiction. C. Yes, Hacksaw's claim is a cross claim and is not required to be asserted in this suit. D. No, the third-party complaint against both Hacksaw and Ginger Paint fails for lack of diversity jurisdiction and must be dismissed.

B. No, Hacksaw's claim is a compulsory counterclaim and will be supported by supplemental jurisdiction.

Pesca, a citizen of New York, sued Dash, a citizen of New Jersey, for $3 million. Pesca brought this suit in federal court in New York and asserts a state law claim for fraud. He claims that Dash, the broker in a real estate sale, falsely induced him to believe the air-conditioning system in the $15 million building in New York that he sold to Pesca was operating efficiently. Dash then impleaded Diamond, a citizen of New York, who had owned the property Dash sold to Pesca. Dash asserts a third-party (impleader) claim for indemnity against Diamond and contends that Diamond gave him incorrect information about the property's air-conditioning system. Dash contends that Diamond therefore is liable to him (Dash) for all or part of the $3 million he might be liable to Pesca. Diamond responded by filing a motion to dismiss this third-party claim by Dash for lack of subject matter jurisdiction. Should the federal court dismiss Dash's third-party claim against Diamond? A. No, because Pesca could have joined Diamond in his federal suit as a defendant. B. No, because both independent and supplemental grounds of subject matter jurisdiction support Dash's claim against Diamond. C. Yes, because supplemental jurisdiction supports only claims by a plaintiff. D. Yes, once Diamond is impleaded complete diversity is destroyed.

B. No, because both independent and supplemental grounds of subject matter jurisdiction support Dash's claim against Diamond.

Walter Bain, a citizen of Texas, sued Sara Cantrell, a citizen of New Mexico, in federal court in Texas based on diversity jurisdiction. Mr. Bain alleges that in 2017 Ms. Cantrell promised to marry him and to come to his home in Dallas for the wedding. Mr. Bain spent over $100,000 in preparation for the wedding and had in addition purchased five first-class tickets for Ms. Cantrell and her family. Five days before the wedding scheduled in Dallas on September 7, 2018, Ms. Cantrell informed Mr. Bain that she did not want to get married and had found another boyfriend with whom she had established a relationship. Mr. Bain was very upset. He fumed and cursed, and then he hired a lawyer to file suit against Ms. Bain in federal court. His lawyer advised him to sue in federal court because Ms. Cantrell would remove the case if it were filed in a Texas state court. The suit was filed on September 12 in the United States District Court for the Northern District of Texas. Mr. Bain had personal service made on Ms. Cantrell two days later. Ms. Cantrell thought this lawsuit was a ploy to force her to marry Mr. Bain, and she did not respond in any way to the suit. In late November, well after the time expired for her to answer, Mr. Bain took a default because of that failure. Shortly thereafter, he moved in federal court to enter a default judgment in the amount of $140,000. The federal court clerk sent notice to Ms. Cantrell, and this time she appeared through her lawyer to contest his motion. Her lawyer pointed out to the judge, with unassailable legal authority, that neither Texas, New Mexico, or the federal government recognizes a cause of action for the breach of an agreement to marry. Mr. Bain's lawyer contends that this defense is too late because it is foreclosed by the entry of default. Should the federal judge proceed to take evidence of damages? A. Yes, because the entry of default has foreclosed the issue of defendant's liability. B. No, because the entry of a default judgment must be based on a properly asserted and valid cause of action. C. Yes, because the federal court in this diversity case can authorize a new cause of action to compensate him for his losses. D. No, because his motion should have been submitted to the federal court clerk for calculation of damages

B. No, because the entry of a default judgment must be based on a properly asserted and valid cause of action.

Beth Pascual, a citizen of Texas, was hurt when a stairway collapsed while she was visiting a friend's apartment in Houston. The apartment complex in which she was injured is owned by Hicks Investment Company (HIC), a Delaware corporation with its principal place of business in New York. HIC owns the Houston apartment complex as an investment, but does no other business in Texas. Ms. Pascual sued HIC in a federal court in Texas seeking $800,000 in damages for her extensive injuries and served HIC's president while he was attending a convention in Dallas. HIC has moved to dismiss, contending the federal court in Texas has no jurisdiction over it. Should the federal court dismiss the case? A. No, because the federal court has transient jurisdiction over HIC. B. No, because the federal court has specific jurisdiction over this case. C. Yes, because ownership of local property can no longer support jurisdiction. D. Yes, because HIC has not had any purposeful contact with Texas.

B. No, because the federal court has specific jurisdiction over this case.

Joan Ash, a Texas citizen, sued Ford Motor Company, Inc., for $500,000 in a federal court in Texas alleging that Ford defectively designed the car that caused her severe injuries in Texas. This products liability claim is a state-created cause of action. Ford is incorporated in Delaware but has its executive offices in Michigan. In addition, three of Ford's six manufacturing plants are in Texas. Ford also earns over $2.5 billion in revenue from car sales in Texas. Ford has moved to dismiss this case for lack of subject matter jurisdiction contending that diversity was destroyed when Ms. Ash sued Ford in Texas where it engages in continuous and systematic business activities. Should the federal court dismiss this suit? A. Yes, diversity would exist if Ms. Ash sued in Michigan but was destroyed when she sued in Texas. B. No, both complete diversity and the requisite jurisdictional amount have been satisfied. C. Yes, diversity does not exist because Ford is "at home" in Texas. D. No, Ford does not engage in a sufficient amount of business in Texas for that state to be its principal place of business.

B. No, both complete diversity and the requisite jurisdictional amount have been satisfied.

Imagine that lowell franklin purchased an insurance policy from the international life insurance company while living in north carolina and that he moved to california only a week before his death. Assume that the beneficiary of the policy, lulu mcgee, had always lived in california. If the insurance conpany had no other connection to california, would a california court have authority to exercise personal jurisdiction over the insurance compnay if the company makes a timely objection? A. No, the california statute does not confer personal jurisdiction in this case B. No, personal jurisdiction in this case would probably be unconstitutional C. Yes, the california statute permits personal jurisdiction in this type of case, because the case involves an insurance policy that covered someone who was a california citizen at the time of death D. Yes, the statute probably applies here, and its application to this case would be constitutional: witnesses regarding the cause of mcgee's death are in california, and california has an interest in ensuring that one of its citizen recovers on an insurance policy. In addition, mrs. mcgee has an interest in suing in california

B. No, personal jurisdiction in this case would probably be unconstitutional

Lucy Bankers was one of the founding members of the non-profit organization known as Free the Net (FTN). This organization was created under the law of Texas, and it included over 200 members in Austin, Texas. In 2016, Ms. Bankers filed a request under the Freedom of Information Act (FOIA), a federal statute. She requested documents from the Federal Communications Commission (FCC) concerning net-neutrality communications between the FCC and Internet service providers. The FCC rejected her request, and Ms. Bankers sued the FCC in federal court pursuant to a federal cause of action. Her claim was based on her allegations that the documents were government papers subject to the Freedom of Information Act. The federal district court in the Western District of Texas, Austin Division granted the FCC a summary judgment, holding that the documents were not subject to the FOIA. In 2018, Sierra Cortez's demand for release of the same documents was rejected by the FCC. As a result, Ms. Cortez asserted in the federal court in Austin the same federal cause of action that Ms. Bankers had asserted previously in that court. The FCC contends that Ms. Bankers and Ms. Cortez are both members of FTN, both live in Austin, both use the same lawyer, and are close friends. The FCC therefore argues, as an affirmative defense, that Ms. Bankers was the virtual representative of Ms. Cortez. The FCC contends therefore that the federal court should enter a summary judgment against Ms. Cortez based on claim preclusion. Should the federal court grant the FCC's motion for summary judgment on either ground? A. Yes, the Supreme Court has held in a similar case that where the facts indicate a close relationship between the two plaintiffs, the first is a virtual representative of the other. B. No, the Supreme Court has held that when the second plaintiff did not have her day in court she cannot be subjected to preclusion because of virtual representation. C. Yes, the federal court should use issue preclusion to prevent Ms. Cortez from re-litigating the same federal claim. D. No, the two claims by these plaintiffs would not satisfy the same-evidence test applied by federal courts.

B. No, the Supreme Court has held that when the second plaintiff did not have her day in court she cannot be subjected to preclusion because of virtual representation.

This question in the Chamberlain case was whether a nine-car string of railcars had collided with the two-car string on which Mr. Chamberlain rode. The plaintiff had to carry her burden of production by offering sufficient evidence to prove the nine-car string did collide with the two-car string to show the responsibility of the Railroad for the death of Mr. Chamberlain. The plaintiff had only Mr. Bainbridge, "then employed by the road" who testified he heard a loud bang and then thought he saw the two strings moving together. In that case, the three brakemen on the nine-car string testified that their cars did not collide with the two-car string. Assume instead that one of those three brakemen, Mr. Sackman, was no longer employed by the railroad. At trial, he testified that though did not see the collision, he heard and felt the crash where he stood at the back of the nine-car string. He also testified that the crash had almost caused him to fall off the nine-car string. Shortly after he righted himself, he saw his car pass over the decedent's body on the tracks. The railroad supports its case with the testimony of the other two brakemen on the nine-car string, who testified that no crash happened. In addition, the two brakemen piloting the seven-car string in front of decedent's two-car string testified they neither heard nor saw any collision between the two-car and the nine-car strings. Assume the Railroad filed its motion for judgment as a matter of law after the close of all the evidence contending that no reasonable jury could find for the plaintiff on the collision issue. The judge concluded that the Railroad's case was supported by the testimony of four eyewitnesses whose testimony clearly outweighed the testimony of the plaintiff's two witnesses (Bainbridge and Sackman). Should the judge have granted the Railroad's motion for judgment as a matter of law? A. Yes, the Railroad had four witnesses arrayed against the plaintiff's two witnesses, and a reasonable jury cannot rule in favor of the two witnesses. B. No, the judge in this case necessarily questioned the credibility of the witnesses and weighed the evidence, which is role of the jury. C. Yes, the judge has experience with matters of evidence and can decide to take the factfinding function away from the jury. D. No, a judge cannot grant judgment as a matter of law before allowing the jury to determine the factual issues.

B. No, the judge in this case necessarily questioned the credibility of the witnesses and weighed the evidence, which is role of the jury.

Blaze Ortega, a citizen of Texas, joined with her mother, Ophelia, also a citizen of Texas, in a suit for damages against the Hunt Company, incorporated in Delaware and having its principal place of business in Illinois. They sued in a federal court in Texas based on diversity jurisdiction. Ms. Ortega sought damages in the amount of $150,000 and her mother sought $25,000 in damages. Both claims arose from the same accident they allege was caused by the defendant's negligence. Ms. Ortega had used a can of diced tomatoes sold by Hunt in making a dish for her dinner. The can contained not only tomatoes but also slivers of metal. When she consumed these metal slivers, she was injured so severely that her mother had to rush her to the hospital. While she was recuperating from surgery, her mother had to quit her job and provide care and money for medications. Hunt filed a motion to dismiss the mother's claim because it failed to satisfy the jurisdictional amount, and, in addition, Hunt contends that the aggregation rules prevent adding claims by multiple parties to satisfy the jurisdictional amount. Should the judge dismiss the mother's claim? A. No, the aggregation rules allow the mother to aggregate her claim with that of her daughter to satisfy the jurisdictional amount. B. No, the mother's claim arises from the same accident that gave rise to the daughter's $150,000 claim, and supplemental jurisdiction therefore supports it. C. Yes, supplemental jurisdiction does not apply to support jurisdictional amount deficiencies. D. Yes, the no-aggregation rule applies to block supplemental jurisdiction over the mother's claim.

B. No, the mother's claim arises from the same accident that gave rise to the daughter's $150,000 claim, and supplemental jurisdiction therefore supports it.

The plaintiff filed a breach of contract cause of action in federal court based on diversity. The defendant had agreed to sell her his land but failed to appear at the closing or to perform the contract. As her first remedy, the plaintiff seeks specific performance of the contract. In her complaint, she has also requested damages for the breach if the court fails to grant specific performance. In a timely fashion, the plaintiff filed a demand for jury trial on all issues in this case. The defendant moved to quash this demand because he contends specific performance is the plaintiff's primary remedy. He argues that the right to this equitable remedy should be tried first without a jury. Specific performance was developed in the equity courts and is based on an injunctive order. The plaintiff contends that she has also sought common law damages, and that may be the only remedy she obtains. All the issues leading to the defendant's liability for breach are common to the specific performance remedy and to the damages remedy. Should the court deny the plaintiff a jury trial on the issues common to the damages and to the specific performance remedy? A. Yes, specific performance is obviously the plaintiff's preferred remedy and therefore no right to a jury exists as to the fact issues leading to liability for that equitable remedy. B. No, the right to a jury trial exists as to all issues necessary for a finding of liability for damages. C. Yes, a judge can deny jury trial on all issues by ruling that the primary remedy is equitable. D. No, in federal court, a party has a right to a jury trial on any fact issue even when the plaintiff seeks only an equitable remedy.

B. No, the right to a jury trial exists as to all issues necessary for a finding of liability for damages.

In a federal diversity action, the federal judge applied a federal judge-made rule that allowed her to comment on the weight of the evidence while instructing the jury. A procedural rule of the state in which this federal court was located forbids a judge from commenting on the weight of the evidence. Was the judge's ruling in this regard an error? A. Yes, in a diversity action the federal judge can never apply a federal judge-made rule. B. No, this federal judge-made rule arguably satisfies the twin-aims test and is part of the judge-jury relationship supported by the Seventh Amendment. C. Yes, no federal judge-made rules can exist. D. No, because a state rule that regulates procedure cannot be applied in a diversity case.

B. No, this federal judge-made rule arguably satisfies the twin-aims test and is part of the judge-jury relationship supported by the Seventh Amendment.

Which one of the following cases can be removed from state to federal court? A. Powers, a citizen of Texas, sues Daniel, a citizen of Texas, in a Texas state court on a state law claim for negligence, but Daniel (the defendant) asserted a federal cause of action as a counterclaim against Powers. B. Powers, a citizen of Utah, sues Damon, a citizen of Utah, on a cause of action authorized by a federal statute in a Utah state court seeking $10,000 in damages. C. Powers, a citizen of Texas, sues Damon, a citizen of Utah, on a state law breach of contract claim in a Utah state court for $200,000. D. Powers, a citizen of Texas, sues Damon, a citizen of Texas, and Dolman, a citizen of Nevada for $200,000 on a state law breach of contract claim in a Utah state court, but 13 months after bringing suit voluntarily dismisses Damon as a defendant.

B. Powers, a citizen of Utah, sues Damon, a citizen of Utah, on a cause of action authorized by a federal statute in a Utah state court seeking $10,000 in damages.

Which one of the following cases can be removed from state to federal court? A. Powers, a citizen of Texas, sues Daniel, a citizen of Texas, in a Texas state court on a state law claim for negligence, but Daniel (the defendant) asserted a federal cause of action as a counterclaim against Powers. B. Powers, a citizen of Utah, sues Damon, a citizen of Utah, on a cause of action authorized by a federal statute in a Utah state court seeking $10,000 in damages. C. Powers, a citizen of Texas, sues Damon, a citizen of Utah, on a state law breach of contract claim in a Utah state court for $200,000. D. Powers, a citizen of Texas, sues Damon, a citizen of Texas, and Dolman, a citizen of Nevada for $200,000 on a state law breach of contract claim in a Utah state court, but 13 months after bringing suit voluntarily dismisses Damon as a defendant.

B. Powers, a citizen of Utah, sues Damon, a citizen of Utah, on a cause of action authorized by a federal statute in a Utah state court seeking $10,000 in damages.

Alice Jackson grew up in Texas but recently enrolled in a two-year nursing program in New Mexico where she lives in her college's dormitory. After graduation she intends to work for her mother, a cardiologist living in El Paso. One year after starting school in New Mexico, she asserted a state medical malpractice cause of action seeking $600,000 damages in a federal district court in Texas. The defendant is a doctor who is a Texas citizen. The doctor filed a motion to dismiss the suit for lack of jurisdiction. Should the federal court dismiss this suit? A. Yes, because diversity jurisdiction cannot be used to support a suit in federal court if the defendant is a citizen of the forum state. B. Yes, because both parties were citizens of Texas when the complaint was filed. C. No, because Ms. Jackson intended to continue attending college in New Mexico for one more year when she filed suit. D. No, because Ms. Jackson's New Mexico residence alone is sufficient to establish her citizenship there even though she always intended to return to Texas.

B. Yes, because both parties were citizens of Texas when the complaint was filed.

The plaintiff is a Texas citizen who traveled from Texas to Arizona in order to reach a store in Phoenix that sells large farming equipment and implements. The plaintiff told the owner of the store that he was a Texan and that he was looking for a small tractor to take back to Texas to cut brush on his ranch. The store owner, a citizen of Arizona, persuaded the plaintiff that for the Texas climate he needed the Capstone Supreme, a $20,000 tractor. The plaintiff bought the Capstone and carried it home to Texas on his flatbed truck. Several months later, while he was washing this tractor in his pasture, its engine started for no discernible reason, and the tractor rolled over the plaintiff causing him severe injuries. After his release from the hospital, the plaintiff sued the owner of the Arizona store in Texas state court seeking $700,000 in damages under Texas tort law. The defendant has challenged the court's personal jurisdiction and seeks dismissal of this suit for that reason. Should the Texas state court dismiss the suit? A. No, the court has general jurisdiction because the defendant induced a known Texas buyer to purchase this allegedly defective tractor. B. Yes, because the Texas court has neither specific nor general jurisdiction over the defendant. C. Yes, because due process protects a defendant against litigation in a state other than his residence unless he is served in the forum personally. D. No, because the defendant put the tractor in the stream of commerce knowing it would end up in Texas.

B. Yes, because the Texas court has neither specific nor general jurisdiction over the defendant.

In the case described above, assume that after Pesca learned of Dash's third-party claim against Diamond, he amended his complaint to dismiss his claim against Dash and to substitute the same state-law fraud claim for $3 million against Diamond. Diamond responded by filing a motion to dismiss this claim asserted against him by Pesca for lack of subject matter jurisdiction. Will the federal court grant this motion? A. No, because Pesca needs no jurisdictional grounds since Diamond was already a party in the suit. B. Yes, because the claim by Pesca against Diamond is not supported by diversity or supplemental jurisdiction. C. Yes, because supplemental jurisdiction cannot exist after the diverse defendant (Dash) was dismissed. D. No, because Pesca's claim against Diamond arises from the same nucleus of operative facts that gave rise to Pesca's claim against Dash.

B. Yes, because the claim by Pesca against Diamond is not supported by diversity or supplemental jurisdiction.

Amir Saladin, is a citizen of Texas, and he brought a wrongful death action against Furniture Movers, Inc. (FMI), incorporated in Delaware with its executive offices in California. He sued seeking $600,000 in damages based on a claim that FMI's driver negligently caused an auto accident in which his wife was killed, and he also demanded a jury trial. During discovery, FMI demanded that Mr. Saladin explain what evidence or witness he had to prove that FMI's driver was negligent. Having received no such information, FMI moved for summary judgment shortly after the end of the discovery period. FMI included with its motion sworn depositions from two eyewitnesses and the deposition testimony by FMI's driver that strongly supported the conclusion that, moments before the crash, Mrs. Saladin pulled out into the path of the FMI truck. Mr. Saladin has submitted no evidence and has named no eyewitness to support the conclusion that FMI's driver was negligent. He contends that the jury is very likely to grant recovery because the accident caused his wife's death and that he will prove negligence at trial through cross-examination of the eyewitnesses. Should the federal court grant the summary judgment on FMI's behalf? A. No, because a summary judgment would effectively deny the plaintiff his constitutional right to a jury trial. B. Yes, because the plaintiff has failed to produce any evidence of the defendant's negligence, which is the basis of his claim. C. No, because it is very likely that a jury would be sufficiently sympathetic to the plaintiff to give him a verdict anyway. D. Yes, because the judge can weigh the evidence and find in favor of the defendant.

B. Yes, because the plaintiff has failed to produce any evidence of the defendant's negligence, which is the basis of his claim.

The widower of a woman who had been killed in an auto accident brought suit in federal court against the corporation that owned the large truck that had been involved in the accident. In this suit, he demanded a jury trial. The widower, Amir Saladin, is a citizen of Texas, and has brought the wrongful death action against Furniture Movers, Inc. (FMI), incorporated in Delaware with its executive offices in California. He brought suit seeking $600,000 in damages based on a claim that FMI's driver negligently caused the accident. During discovery, FMI demanded that Mr. Saladin explain what evidence or witness he had to prove that FMI's driver was negligent. Having received no such information, FMI moved for summary judgment shortly after the end of the discovery period. FMI included with its motion sworn depositions from two eyewitnesses and the deposition testimony by FMI's driver that strongly supported the conclusion that, moments before the crash, Mrs. Saladin pulled out into the path of the FMI truck. Mr. Saladin has submitted no evidence and has named no eyewitness to support the conclusion that FMI's driver was negligent. He contends that the jury is very likely to grant recovery because the accident caused his wife's death and that he will prove negligence at trial through cross-examination of the eyewitnesses. Should the federal court grant the summary judgment on FMI's behalf? A. No, because a summary judgment would effectively deny the plaintiff his constitutional right to a jury trial. B. Yes, because the plaintiff has failed to produce any evidence of the defendant's negligence, which is the basis of his claim. C. No, because it is very likely that a jury would be sufficiently sympathetic to the plaintiff to give him a verdict anyway. D. Yes, because the defendant's evidence outweighs that of the plaintiff.

B. Yes, because the plaintiff has failed to produce any evidence of the defendant's negligence, which is the basis of his claim.

In March of 2020, Ben Cullen, who is a citizen of Texas, was injured when he suffered three minor cuts when the power drill he was using threw off sharp metal shards. He was using this drill while working in Austin, Texas, for a construction company. The company allowed Mr. Cullen to take the day off and advised him to see his doctor. His doctor cleaned and bandaged the cuts, treated them with Neosporin ointment to ward off infection, and provided Mr. Cullen with Motrin, an anti-inflammatory pain reducer. He continued working with the drill the next day and has had no further difficulty because of the incident. Mr. Cullen's nephew, who has recently been licensed to practice law, convinced him to bring a tort suit for defective design against Adarand Tools, Inc. Adarand manufactures the drill which caused Mr. Cullen's injuries and it sold this drill to Mr. Cullen's employer because of its advertising in Texas. The nephew filed this suit in the United States District Court for the Western District of Texas, Austin division, and has alleged that the amount in controversy exceeds $75,000. He therefore contends that this federal court has diversity jurisdiction over the case. Adarand is incorporated in Delaware and its executive headquarters is inCalifornia, and it has moved to dismiss this suit in federal court. Should the federal court dismiss this suit? A. No, the jurisdictional amount is satisfied because at the time of filing this complaint, Mr. Cullen made a sufficient pleading to that effect. B. Yes, no reasonable jury could award Mr. Cullen an amount approaching $75,000, and the judge would find the amount insufficient as a matter of law. C. No, in a tort suit, a federal court will always assume that $75,000 is a reasonable award. D. Yes, Adarand is not subject to jurisdiction in Texas by any court.

B. Yes, no reasonable jury could award Mr. Cullen an amount approaching $75,000, and the judge would find the amount insufficient as a matter of law.

Charles Omega, a citizen of Oklahoma, brought suit in a Texas state court against Exxon Mobil, which has an oil & gas lease on his land in Texas. Exxon is incorporated in Missouri and has its principal place of business in Texas, and for that reason could not remove the case to federal court. In this suit, Mr. Omega asserted a Texas unjust-enrichment claim arising from Exxon's underpayment of royalties owed him under the lease in the amount of $200,000. The Texas trial court gave Exxon a summary judgment, holding that Mr. Omega could not recover under an unjust-enrichment claim when the same conflict between him and Exxon was based on a valid contract. ​Instead of appealing the Texas judgment, Mr. Omega filed a breach of contract claim against Exxon in an Oklahoma state court seeking $200,000 in damages for underpayment of royalties under the lease. Exxon filed an answer in the Oklahoma court asserting the affirmative defense of claim preclusion and moved for a dismissal with prejudice on that basis. The only issue in regard to the application of claim preclusion is whether these two causes of action are the "same claim." Oklahoma uses the same-evidence test to determine whether a claim in the second suit is the same claim as that of the first suit. Mr. Omega notes that the evidence that would be introduced to prove an unjust enrichment claim is very different from the evidence needed to prove a breach of contract claim. Texas applies the "transaction test" and would give its judgment claim-preclusive effect. Should the Oklahoma court grant Exxon's motion for dismissal? A. No, the Oklahoma court must apply Oklahoma claim-preclusion law and will hold that the evidence necessary for the two claims is different. B. Yes, the Oklahoma court will apply the law of Texas, and both claims arise from the same transaction or series of transactions. C. No, the Oklahoma court will find that the two claims asserted by Mr. Omega arise from independent primary rights and, therefore, are not the same claim. D. Yes, the Oklahoma court will find that the same-evidence test applies and that the evidence needed for both claims is sufficiently similar.

B. Yes, the Oklahoma court will apply the law of Texas, and both claims arise from the same transaction or series of transactions.

In May of 2020, a Greyhound bus ran off an overpass on Loop 410 in San Antonio. The accident caused serious injuries to five passengers, all of whom are citizens of Texas residing in San Antonio. All the passengers are represented by the same San Antonio lawyer. Greyhound is a Delaware corporation with its principal place of business in Oregon. One of the passengers sued in the federal court in San Antonio against Greyhound. In the trial of this case, the jury held that the accident was caused by a defect in the highway and that the bus company was not negligent. After this judgment was entered, another passenger sued Greyhound in the same federal court. When this plaintiff's case reached trial, the jury ruled that Greyhound's negligence (not the highway defect) caused the accident. One month after judgment was entered in this second case, the third passenger sued in the same federal court claiming damages because of Greyhound's negligence in causing the accident. This plaintiff asserts that Greyhound cannot contest the issue of its negligence because that issue has been adjudicated in the second suit. Should the federal court use its discretion to require re-litigation of the issue of Greyhound's negligence? A. No, Greyhound has had its day in court on the issue of negligence and is barred by claim preclusion from re-litigating its negligence. B. Yes, this case is an example of the problems of allowing the offensive use of non-mutual use of issue preclusion. C. No, Greyhound can use the judgment in the first suit to bar re-litigation by the third plaintiff. D. Yes, Greyhound should be able to re-litigate the issue of its negligence because it has not had its day in court on that issue.

B. Yes, this case is an example of the problems of allowing the offensive use of non-mutual use of issue preclusion.

Under federal patent law, Apex Company has acquired a patent covering its widgets. Allied Manufacturing Company entered a contract with Apex to manufacture Apex's widgets. In the contract, Allied agreed to pay Apex $1,000 for each widget it manufactures and sells. Under which one of the following circumstances could Apex successfully invoke federal question jurisdiction? A. Allied fails to pay and Apex sues it to obtain the money it should have received under the contract. B. Allied fails to pay and Apex sues it to obtain tort damages because of the harm caused Apex's reputation. C. Allied fails to pay and Apex successfully cancels the contract and sues Allied for patent infringement. D. Allied fails to pay and Apex sues for specific performance of the contract.

C. Allied fails to pay and Apex successfully cancels the contract and sues Allied for patent infringement.

In 2018, Martha Monroe, a citizen of Iowa, brought a wrongful-death state tort action for the death of her husband. She filed this suit in federal court in Nebraska against a utility company. She sued the utility company for $2 million. That company is incorporated and has its principal place of business in Nebraska. The utility company then impleaded Carter Construction Company. It contended that Carter was a joint tortfeasor in causing the death of Ms. Monroe's husband and was therefore liable to the utility company for any amount it was forced to pay Mrs. Monroe. Carter is incorporated in Nebraska but has its principal place of business in Iowa. Which one of the following is a correct statement? A. The federal court has federal question jurisdiction to adjudicate this suit. B. Once joined by the utility company, Carter's citizenship in Iowa requires the federal court to dismiss the suit. C. Diversity jurisdiction over Mrs. Monroe's suit is determined at the time she filed her original complaint, and the impleader of Carter is supported by supplemental jurisdiction. D. The federal court in its discretion can adjudicate any claim asserted against a third-party defendant.

C. Diversity jurisdiction over Mrs. Monroe's suit is determined at the time she filed her original complaint, and the impleader of Carter is supported by supplemental jurisdiction.

In which one of the following cases is the amount-in-controversy requirement met to allow adjudication of all claims in federal court? (Assume the complete diversity requirement is satisfied.) A. Grenardo sues Teeter seeking $50,000 for losses he suffered in a business deal and Cochran joins as co-plaintiff seeking $60,000 from Teeter for his losses in the same deal. B. Grenardo sues Teeter seeking $80,000 for injuries suffered in an auto accident, but Cochran joins as a co-plaintiff seeking $50,000 from Teeter for the damages he suffered because of an unrelated breach of contract by Teeter. C. Grenardo sues Teeter seeking $50,000 for a slander cause of action and seeks another $40,000 because Teeter also intentionally struck him with an iPhone. D. Grenardo sues Teeter seeking $50,000 for losses he suffered because of Teeter'sbreach of a business contract and joins Kauffman as a co-defendant seeking $70,000 from him for the losses he suffered in the same deal.

C. Grenardo sues Teeter seeking $50,000 for a slander cause of action and seeks another $40,000 because Teeter also intentionally struck him with an iPhone.

Which of the following cases can be removed to federal court? A. Martinez, from Texas, sues Murphy, from Utah, on a state law breach of contract claim, in a Utah state court, for $200,000 B. Martinez, from Texas, sues Murphy, from Utah, and Mercer, from Nevada, on a state law breach of contract claim, in a Utah state court, for $200,000 C. Martinez, from Texas, sues Murphy, from Utah, on a claim arising under federal law. She brings the suit in a Utah state court. D. Martinez, from Texas, sues Hawkins, from Texas, on a state law claim for negligence. Hawkins counterclaims against Martinez for violation of a federal statute.

C. Martinez, from Texas, sues Murphy, from Utah, on a claim arising under federal law. She brings the suit in a Utah state court.

Assume Congress has passed a federal statute that bars any railroad from giving reduced prices for passage. This federal statute does not create a cause of action because Congress chose to have it enforced by the Federal Trade Commission. Union Pacific Railroad, incorporated in and having its principal place of business in Texas, reacted to this statute by halting performance of its contract with Megan Jolley, a citizen of Texas, under which it had promised her half-price tickets for life. Megan had performed tax services for the railroad, and the railroad promised to give her reduced price railroad tickets instead of paying her a salary. Megan sues the railroad in federal court for not honoring its contract, claiming the federal statute either doesn't apply to their contract or, if it does, it deprives her of property in violation of the Due Process Clause. Which one of the following is a correct statement? A. Megan can invoke federal question jurisdiction because her claim depends on issues of federal law sufficient under Grable. B. Megan can bring an action in federal court whenever a disputed issue requires reference to federal law. C. Megan cannot bring an action in federal court because her complaint fails to assert a federal claim, and the elements of her state cause of action do not require decision on a substantial federal issue. D. Megan can bring an action in federal court because the railroad's statutory defense will raise an important constitutional issue.

C. Megan cannot bring an action in federal court because her complaint fails to assert a federal claim, and the elements of her state cause of action do not require decision on a substantial federal issue.

The plaintiff filed this complaint in a federal district court in California seeking damages in excess of $75,000. The suit was filed and served on the defendant on the last day before the running of the applicable statute of limitations. The plaintiff alleges that while she was living in Illinois the defendant, who is domiciled in Illinois, shot off a bottle rocket that set her house in Chicago on fire. After the fire destroyed her home and caused her serious burns, she moved back to live with her mother in California and then filed this suit. Thirty days after the statute of limitations expired, the defendant, who has never set foot in California, moved to dismiss this suit. If you represented the plaintiff, how would you respond to this motion? A. Contend that the federal court in California has specific jurisdiction over the defendant. B. Contend that the federal court in California has general jurisdiction over the defendant. C. Move for transfer of the case in the interest of justice to the federal court in Illinois in which it could have been brought. D. Contend that the federal court in California has federal-question jurisdiction over the defendant.

C. Move for transfer of the case in the interest of justice to the federal court in Illinois in which it could have been brought.

Jill Risen filed this complaint in a federal district court in Texas seeking damages of more than $75,000 against Portia Wells. Ms. Risen is a citizen of Texas, and Ms. Wells is a citizen of Iowa. Ms. Risen had initiated negotiations and signed a contract in Iowa with Ms. Wells in which she agreed to secure a patent on an invention created by Ms. Wells. Ms. Risen secured the patent, but Ms. Wells refused to pay. After several years during which the parties sought to reach a settlement, Ms. Risen filed this suit. Ms. Wells has moved to dismiss the suit for lack of personal jurisdiction. Unfortunately, Ms. Risen does not believe she can sustain jurisdiction in Texas and now realizes that Ms. Wells has stalled this matter until the applicable statute of limitations has expired. A dismissal will therefore subject Ms. Risen's cause of action to a limitations defense when she refiles it. If you represented Ms. Risen, how would you respond to the motion to dismiss? A. Ms. Risen should argue that she has satisfied all the requirements for diversity jurisdiction in this federal court. B. Ms. Risen should argue that the Texas federal court has general jurisdiction over the defendant. C. Ms. Risen should move for transfer of the case in the interest of justice to the federal court in Iowa. D. Ms. Risen should respond with a motion to remand this case to state court.

C. Ms. Risen should move for transfer of the case in the interest of justice to the federal court in Iowa.

Jane Paggett, a Texas citizen, sells custom speedboats to residents of California and Florida. In January 2018, she contacted Helmut Daniels, a Californian who is domiciled in San Francisco, and arranged to sell him a $850,000 speedboat. After delivering the boat to him in California, she demanded payment, but Daniels has continued to stall her. In August 2018, Paggett learned that Daniels has for many years owned a vacation house outside Las Vegas, Nevada, where he resides while gambling in Las Vegas. She therefore filed suit against Daniels on August 21 in a Nevada state court seeking $850,000, plus interest, for breach of contract. In September 2018, she had Daniels personally served with process in a Las Vegas gambling casino. Daniels has moved to dismiss the suit. Should the Nevada state court dismiss this case? A. Yes, because the Nevada court does not have specific jurisdiction over Daniels. B. Yes, because Daniels is not domiciled in Nevada. C. No, because Daniels was personally served in the forum state. D. No, because Daniels' vacation house in Nevada gives a court of that state general jurisdiction over him.

C. No, because Daniels was personally served in the forum state.

In 2005, while living in Indiana, Joe Gail purchased a $20,000 life insurance policy from the Good-Bye Life Insurance Co. (GBL), which is incorporated and has its principal place of business in Nevada. GBL had used its Internet program to send email ads to numerous people in Indiana, and Mr. Gail saw and responded to the ad sent to him by taking out a life insurance policy with GBL. In 2018, Mr. Gail moved to Texas to live with his mother because he was suffering from an illness. He continued to make payments on the insurance policy from Texas, and his checks were received in Nevada and cashed by GBL. In 2020, Mr. Gail died after a long stay in the hospital. His mother, Betty Gail (the beneficiary under his policy), requested payment from GBL. GBL refused to pay her claim, contending that Mr. Gail committed suicide, which violates a contractual limitation. Ms. Gail sued the company for breach of contract in a state court in Texas seeking $20,000. GBL has filed the appropriate papers seeking to dismiss this suit. Attached to its motion, GBL filed affidavits showing that Gail was the only insured it has in Texas, that it has no office or employee in Texas, and that it has never advertised in any manner for business in the state. Does the Texas court have jurisdiction over this suit? A. No, because the jurisdictional amount has not been satisfied. B. Yes, because GBL had a purposeful contact by continuing to insure Mr. Gail's life after he moved to Texas. C. No, because GBL's relationship with Texas came about because of the unilateral action of Mr. Gail and not because of any GBL contacts. D. Yes, because under the Court's recent decision in Ford Motor Co., it is no longer necessary for the defendant to have a purposeful contact with the forum.

C. No, because GBL's relationship with Texas came about because of the unilateral action of Mr. Gail and not because of any GBL contacts.

Jack Slick was born and raised in Texas, but from 2015 to 2018 he lived in Maine and then for a year in Connecticut. In 2018, he moved to Vermont and took a job working on a farm for the harvesting season. When the harvesting season was over, the owner of the farm gave him permanent employment because he liked Mr. Slick's working habits. Mr. Slick did tell the farmer that at some point he hoped to reconcile with his girlfriend and go back to live with her in Connecticut, but he continued to live in the farmer's garage apartment. In 2021, while driving the farmer's tractor on a nearby country road in Vermont, Mr. Slick was struck by a speeding car driven by Hack Bund. Mr. Slick sued Mr. Bund in federal court in Vermont based on a state law tort claim for negligence. He alleged in his complaint that he is a citizen of Texas, that Mr. Bund is a citizen of Vermont, and that the matter in controversy is of a sum or value in excess of $75,000. Mr. Bund filed his answer, and one of his defenses was based on the contention that this federal court lacked subject matter jurisdiction. Will the federal court grant this motion? A. Yes, because Mr. Slick's current domicile is in Vermont. B. No, because Mr. Bund has waived his objection to subject matter jurisdiction by failing to file a preanswer motion. C. No, because Mr. Slick has retained his Texas domicile. D. Yes, because Mr. Slick's wandering ways means he has no domicile.

C. No, because Mr. Slick has retained his Texas domicile.

Dash Roberts, a domiciliary of San Antonio sells farm equipment. In 2020, he sold a tractor to Rubik Haulsburg, who lives on a farm outside San Antonio. Mr. Haulsburg paid 10% down and was supposed to pay 10% each month thereafter until the total cost was paid. However, he made only that first payment and no others. Mr. Roberts filed suit in Texas state court in San Antonio seeking $25,000 or the return of the tractor. He gave the district court clerk Mr. Haulsburg's mailing address and had the clerk's office serve him with process by first class mail. Mr. Haulsburg never responded to this suit, and Mr. Roberts took a default judgment for $25,000. When he sought to execute this judgment on Mr. Haulsburg's farm, Mr. Haulsburg finally responded with a motion to set aside the default judgment. He contends that he has never received notice of the suit and therefore has been deprived of the due process of law. Evidently, Mr. Haulsburg's mailbox is on the road that runs in front of his farm, but squirrels and other marauding mammals often steal mail from these isolated mailboxes. Mr. Haulsburg contends the court must set aside the default judgment because he did not receive actual notice of the suit. The evidence supports Mr. Haulsburg's contention that he did not receive the clerk's notice. Must the court set aside the default judgment for violation of the Due Process Clause? A. Yes, because service by first class mail is inherently inefficient and personal service is more likely to provide defendants with actual notice. B. Yes, because the court would violate the Due Process Clause by enforcing a default judgment against someone who did not receive actual notice. C. No, because the Due Process Clause requires a state to use an efficient means of giving actual notice, and first-class mail has been held to be such. D. No, because the Due Process Clause does not protect those who take such poor care of their mailboxes.

C. No, because the Due Process Clause requires a state to use an efficient means of giving actual notice, and first-class mail has been held to be such.

Josh Collins, a domiciliary of San Antonio, Texas, sells farm equipment. In 2017, he sold a tractor to Henrik Denman, who lives on a farm outside San Antonio. Mr. Denman paid 10% down and was supposed to pay 10% each month thereafter until the total cost was paid. However, he made only that first payment and no others. Mr. Collins filed suit in Texas state court in San Antonio seeking $25,000 or the return of the tractor. He gave the district court clerk Mr. Denman's address and had the clerk's office serve him with process by first class mail. Mr. Denman never responded to this suit, and Mr. Collins took a default judgment for $25,000. When he sought to execute this judgment on Mr. Denman's farm, Mr. Denman finally responded with a motion to set aside the default judgment. He contends that he has never received notice of the suit and therefore has been deprived of the due process of law. Evidently, Mr. Denman's mailbox is on the road that runs in front of his farm, but teenagers often steal mail from these isolated mailboxes. Mr. Denman's contends therefore that the court must set aside the default judgment because it is constitutionally deficient. Must the court set aside the default judgment for violation of the Due Process Clause? A. Yes, because service by first class mail is inherently inefficient and personal service is more likely to provide defendant's actual notice. B. Yes, because the court would violate the Due Process Clause by enforcing a default judgment against someone who did not receive actual notice. C. No, because the Due Process Clause requires a state to use an efficient means of giving actual notice, and first-class mail has been held to be such. D. No, because the Due Process Clause requires personal service of process.

C. No, because the Due Process Clause requires a state to use an efficient means of giving actual notice, and first-class mail has been held to be such.

Jack Bell decided to take a tour that would take him in his new Audi sports car from his home in Pennsylvania to Maine. At one point, Mr. Bell failed to pay attention to his map and took a wrong turn. After an hour, he learned that he had driven into Vermont not Maine. As he was turning his car around, he rammed into a car driven by Jane Stokes. Ms. Stokes was seriously injured in this accident in Vermont, and sued Mr. Bell in a Vermont state court. Vermont's long-arm statute allows the assertion of jurisdiction by its courts to the extent allowed by the Due Process Clause. After being personally served in Pennsylvania, Mr. Bell filed a motion to dismiss this suit because, he alleges, the Vermont court does not have personal jurisdiction over him. Should the Vermont court dismiss? A. Yes, the reasonableness factors will deprive the court of jurisdiction even though Mr. Bell has had purposeful contacts with that state. B. Yes, Mr. Bell drove into Vermont by mistake and therefore had no purposeful contacts with the state. C. No, because the case arises out of Mr. Bell's contacts in Vermont. D. Yes, because Mr. Bell was not served in Vermont.

C. No, because the case arises out of Mr. Bell's contacts in Vermont.

While at her home in Carter Lake, Iowa, Louise Blake, a teenager, swallowed a sliver of glass that was contained in the Hunt's Tomato Soup she was eating. This glass fragment caused her serious throat and stomach injuries that required hospitalization. After her recovery, she sued ConAgra Foods, Inc., (owner of Hunt's) in federal court in Iowa for $400,000 in damages based on a state-law cause of action. Her mother joined as a plaintiff in Ms. Blake's suit pursuant to Federal Rule 20, but her mother seeks only $40,000 in damages for her emotional distress caused by her daughter's injury. The mother's claim is also a state-law claim. Ms. Blake and her mother are domiciled in Iowa, and ConAgra Foods, Inc., is incorporated in Delaware and has its executive headquarters in Nebraska. ConAgra owns Hunt's food plants in Indiana, Iowa (which produced the tainted soup), and Nebraska. ConAgra's Iowa plant has a few more employees and generates approximately the same revenue as the Indiana and Nebraska plants. ConAgra has moved to dismiss both of the plaintiffs' claims for lack of subject matter jurisdiction. Should the federal court dismiss either Ms. Blake's or her mother's claim? A. Yes, it should dismiss the mother's claim because a federal court cannot hear any claim worth less than $75,000. B. Yes, it should dismiss both claims because ConAgra is not "at home" in Iowa. C. No, because the federal court has diversity jurisdiction over Ms. Blake's claim and supplemental jurisdiction over her mother's claim. D. No, because Ms. Blake's and her mother's claims can be aggregated to equal a sum in excess of $75,000.

C. No, because the federal court has diversity jurisdiction over Ms. Blake's claim and supplemental jurisdiction over her mother's claim.

In 2017, Mrs. Monroe, a citizen of Iowa, brought a wrongful-death, tort suit for the death of her husband seeking $300,000 in damages in a federal court in Nebraska. She brought the action solely on the grounds of diversity jurisdiction against a utility company, Nebraska Electric, because her husband was killed while working on its high-voltage lines. Nebraska Electric is incorporated and has its principal place of business in Nebraska. That company then impleaded under Federal Rule 14 the Carter Construction Company. Nebraska Electric contends that Carter employed Mr. Monroe and was in charge of the repairs to its lines. Carter is therefore at least a joint tortfeasor in causing the death of Mrs. Monroe's husband. Nebraska law authorizes a cause of action for contribution between joint tortfeasors. However, Carter is incorporated in Nebraska, but has its principal place of business in Iowa. Carter has filed a motion to dismiss the whole of this suit because it has the same citizenship as Mrs. Monroe. Should the federal court grant this motion? A. Yes, the federal court does not have diversity jurisdiction because Iowa citizenship appears on both sides of the litigation. B. Yes, Rule 14 allows the utility to file a closely related claim directly against Carter, but the court lacks either independent or supplemental jurisdiction to hear such a claim. C. No, complete diversity was established when Mrs. Monroe filed her original complaint against Nebraska Electric. D. No, a federal court has discretion to adjudicate a claim when complete diversity does not exist.

C. No, complete diversity was established when Mrs. Monroe filed her original complaint against Nebraska Electric.

Tom Gobert, a citizen of Texas, filed suit in federal court in Texas. He alleged that his employment was wrongfully terminated on December 21, 2020, by his then-employer, a corporation known as Biscuit Air Conditioning (BAC). Mr. Gobert asserted a claim for breach of contract and an additional claim under the federal statute that protects against age discrimination. The plaintiff is currently 63 years old. He alleges that BAC fired him on a Friday and hired his replacement on the following Monday. His replacement is the 25-year-old nephew of the company's president, who recently flunked out of college. Mr. Gobert had been working under a written contract that promised he would be employed for an additional 3 years at a salary of $150,000 per year. He had been employed by BAC under similar contracts for the last 20 years. He seeks damages against the corporation, which is incorporated in Delaware but has its only office in Texas. At trial in federal court, Mr. Gobert recovered $400,000 for his breach of contract claim and $0 for his age discrimination claim. The jury found the evidence insufficient to support any damages under the age discrimination claim. After the judge entered a judgment for $400,000 solely on the breach of contract claim, BAC moved to dismiss the breach of contract claim and vacate the judgment for lack of subject matter jurisdiction. It asserted that this judgment was based on a state law claim between nondiverse parties and thus was invalid. Should the court dismiss this suit? A. No, the defendant waived its objection to a lack of subject matter jurisdiction by failing to make it motion before judgment was entered. B. Yes, after the federal claim failed before the jury no ground of jurisdiction existed to support the judgment. C. No, the court had federal question jurisdiction over the federal claim and supplemental jurisdiction over the state claim because it arose from the same operative facts. D. Yes, supplemental jurisdiction can only exist if the federal claim also succeeded at trial.

C. No, the court had federal question jurisdiction over the federal claim and supplemental jurisdiction over the state claim because it arose from the same operative facts.

Horst Banner lost his Texas land, worth $500,000, through an Internal Revenue Service (IRS) tax sale. He had failed to pay his income taxes for a number of years. The IRS gave him notice by certified mail that it would take his Texas property and sell it through a tax sale. Although he received actual notice of the tax sale, he failed to redeem the property by paying off the taxes. At the tax sale, Tommy Carson paid the taxes and thereby gained legal title to the property. Mr. Banner subsequently discovered a federal statute that appears to require the IRS to give any tax delinquent personal service of process before holding a tax sale. In other words, the statute seems to say that the certified mail service given by the IRS was insufficient. Mr. Banner, a citizen of Arizona, sued Mr. Carson, a citizen of Texas, in a Texas state court based on a state-law, quiet-title action. One element of this state cause of action requires Mr. Banner to prove that Mr. Carson has a defective title. Mr. Banner bases his claim on the argument that Mr. Carson's title is defective based solely on the issue of whether the federal statute required the IRS to give Mr. Banner personal service. The IRS is not a party. Mr. Carson removed this case to federal court by contending that the federal court has both diversity and federal question jurisdiction over this case. Mr. Banner filed a motion to remand the case to Texas state court. Should the federal court grant Mr. Banner's motion to remand? A. Yes, the federal court cannot exercise jurisdiction under either diversity or federal question jurisdiction because Mr. Banner's claim arises under state law. B. No, the federal court can exercise diversity jurisdiction over this case upon removal by Mr. Carson. C. No, the federal court can exercise federal question jurisdiction because Mr. Banner's state claim satisfies the Grable exception. D. Yes, the federal court cannot exercise any form of subject matter jurisdiction because Mr. Carson is a citizen of the forum state.

C. No, the federal court can exercise federal question jurisdiction because Mr. Banner's state claim satisfies the Grable exception.

In 2018, Congress passed a federal statute that bars any railroad from giving reduced prices for passage. This federal statute does not create a cause of action. Union Pacific Railroad, incorporated in and having its principal place of business in Texas, reacted to this statute by halting performance of its contract with Megan Jolley under which it had promised her half-price tickets for life. Ms. Jolley had performed tax services for the railroad while she was domiciled in Texas. The railroad had promised her reduced-price passage instead of paying her a fee for the services she provided it for over two years. In early 2018, she moved to New York to take a job with an accounting firm in that state. After her move and the enactment of the federal statute, Ms. Jolley sued the railroad in federal court in Texas for not honoring its contract and for causing her damages of $100,000. She claims the federal statute either doesn't apply to their contract or, if it does, deprives her of property in violation of the Due Process Clause. Which one of the following is a correct statement? A. Ms. Jolley can invoke federal question jurisdiction because her breach of contract claim depends on issues of federal law sufficient under Grable. B. Ms. Jolley can bring an action in federal court whenever a disputed issue requires reference to federal law. C. The federal court has diversity jurisdiction to adjudicate this suit. D. Ms. Jolley can bring this action based on federal question jurisdiction because the railroad's statutory defense will raise an important constitutional issue.

C. The federal court has diversity jurisdiction to adjudicate this suit.

In 2020, Bella Sims, a citizen of New York, entered a contract with Harmony, Inc., a corporation incorporated in Delaware. Ms. Sims had contacted Harmony's executive offices, which are in New Jersey, offering to create a secure computer system at those executive offices. Harmony does not do business or have an office in New York. Harmony accepted this offer and in its contract with Ms. Sims agreed to pay her $80,000 for the system. Early in 2021, Ms. Sims completed the new system in New Jersey, but Harmony has refused to pay her. It contends that the computer system she installed in its New Jersey offices had a substantial defect. She has now filed a diversity action in federal court in New York complaining of this breach of contract. She had one of Harmony's lower-level employees served with process while he vacationed in New York. Harmony responded to her complaint with a preanswer motion to dismiss for insufficiency of service because this employee was not an officer, general manager, or agent of the corporation. The company's motion also states that it reserves the right to assert any other basis for dismissal if the court denies its motion. The federal judge denied Harmony's preanswer motion and ordered it to file an answer. In that answer, Harmony contends that the New York federal court lacks personal jurisdiction over it. It also alleges that Ms. Sims initiated this contractual relationship by contacting its New Jersey office and there negotiating the contract. Harmony subsequently filed a motion in which it argued that this defense required dismissal of the suit in New York for lack of personal jurisdiction. How should the New York federal judge rule on Harmony's objection to personal jurisdiction? A. The judge should dismiss the suit because specific jurisdiction does not support the New York federal court's personal jurisdiction over Harmony. B. The judge should retain jurisdiction over the suit because even though Harmony did not have any purposeful contacts with New York, the reasonableness factors supply personal jurisdiction in this case. C. The judge should retain jurisdiction because Harmony cannot now challenge the New York federal court's personal jurisdiction. D. The judge should dismiss the suit because suits against Harmony can only be brought in its state of incorporation or in the state in which it has its principal place of business.

C. The judge should retain jurisdiction because Harmony cannot now challenge the New York federal court's personal jurisdiction.

Harvey Sneed, a citizen of Texas, was severely injured when he slid down a water slide into his country club's pool. The water slide was marked as being the product of Pool Slides, Inc., incorporated and having its principal place of business in New Jersey. The country club confirmed that it had ordered a Pool Slides, Inc. product. Sneed sued in a federal court in Texas and contended that the slide was constructed by Pool Slides, Inc., and was defectively designed. After service of process, three insurance companies checked the slide and identified it as having been produced by Pool Slides, Inc., of New Jersey. The defendant, Pool Slides, Inc., admitted in its answer that the slide was its product. However, after the statute of limitations had run on this tort claim, Billy Eagle, the president of Pool Slides visited the Texas country club and examined the slide. He concluded that it was not manufactured by Pool Slides, and the company thereafter filed a motion to amend its answer to include a denial that it produced the slide in question. Should the federal court grant this motion? A. Yes, because Pool Slides has a right to amend its pleading once as a matter of course. B. No, because Pool Slides can only amend its pleadings before the statute of limitations had expired. C. Yes, because Pool Slides did not engage in undue delay or bad faith, and amendments should be freely given. D. No, because granting this motion would unduly prejudice the plaintiff.

C. Yes, because Pool Slides did not engage in undue delay or bad faith, and amendments should be freely given.

While deplaning in Houston, Texas, Gary Alvarez was injured on a United Airlines flight when a heavy briefcase fell out of an overhead compartment and caused him to suffer a concussion. After his recovery, he entered a contract with United in which he agreed to release any tort claim he might have against the airlines. In exchange, United agreed to grant him one free first-class ticket on any flight within the United States each year for the next 15 years. Mr. Alvarez is domiciled in New York. United Airlines does business in every state but is incorporated in Delaware and has established its executive headquarters in New York. Six years after this contract was entered, Congress passed a law requiring any airline operating in the United States to charge a minimum of $500 for a first-class ticket. The ostensible reason for this law was to prevent airlines from giving elected officials lower prices as a means of hiding campaign contributions. On the effective date of this federal statute, United notified Mr. Alvarez that it would no longer honor its contract. He sued United in federal court in New York for $50,000, asserting that United had breached its contract and that the new federal statute is either inapplicable or cannot constitutionally apply to this contract. United Airlines has moved to dismiss the suit in federal court. Should the federal court dismiss the suit? A. No, because Mr. Alvarez's case satisfies the Grable exception as described in the Gunn decision. B. No, because Mr. Alvarez case will be decided solely by reference to federal law. C. Yes, because diversity does not exist and Mr. Alvarez 's complaint alleged a state claim that mentions federal law only in anticipation of a defense. D. Yes, because no court in New York has personal jurisdiction over United in this case.

C. Yes, because diversity does not exist and Mr. Alvarez 's complaint alleged a state claim that mentions federal law only in anticipation of a defense.

Susan Wong, a citizen of Utah, brought a wrongful death action in a federal court in Utah against Zimmer Tire, Inc., which is incorporated in Delaware and has its executive headquarters in California. Zimmer is the parent corporation of Flux Tires, Inc., incorporated and having its principal place of business in Germany. Ms. Wong's husband was killed in Germany when a Flux tire on his rented car lost its tread and caused the car to run into a concrete post. She seeks $2.5 million in damages against both Zimmer and Flux. Zimmer does significant business throughout the United States, including in Utah. The subsidiary conducts no business and has no employees or bank accounts in Utah. Flux manufactures its tires for the European market, but Zimmer distributes approximately 200 of Flux's tires in Utah. The defendant corporations have moved to dismiss for lack of jurisdiction. Should the court dismiss this suit against both defendants? A. No, specific jurisdiction over both exists because 200 of Flux's tires were distributed by Zimmer in Utah. B. No, because general personal jurisdiction exists over Zimmer in Utah. C. Yes, because neither specific nor general jurisdiction exists over either defendant. D. Yes, because the federal court has no subject matter jurisdiction to hear a case against a foreign corporation.

C. Yes, because neither specific nor general jurisdiction exists over either defendant.

Mr. Klinger had been driving at night in Texas where he lived and went to sleep at the wheel. His car swerved off the road, but he was able to stop on the shoulder of the road. Ms. Bridges had been driving her sports car after having gone to several dance clubs. She was behind Mr. Klinger's car, and she noticed that the car in front of her swerved off the road. Although she was not legally intoxicated, she thought the car in front of her had swerved to miss some object in the road. She therefore swerved in her car to miss any object in the road. However, in doing so, she drove onto the shoulder of the road at high speech and crashed her car into Mr. Klinger's car The parties were unable to reach any compromise or settlement. After having been injured in the car crash, Ms. Bridges went home to San Francisco, California, to live with father and to recuperate. She subsequently filed suit in a federal court in Texas seeking $400,000 in damages. In her complaint, she made factual allegations of her injuries, but merely stated that Mr. Klinger negligently caused those injuries. Mr. Klinger filed a pre-answer motion to dismiss her claim for failure to state a claim upon which relief could be granted. Ms. Bridges' lawyer contended that Texas law allowed conclusory allegations of negligence. Mr. Klinger's lawyer countered by arguing that in federal court a plaintiff had to make factual allegations supporting each material fact necessary to support each of the elements of her claim. Should the federal court grant the motion to dismiss in this case? A. Yes, because in diversity cases a federal court should apply the procedural law of the state in which it is located. B. No, because one can always choose to make conclusory allegations if those allegations alert the defendant about the nature of the claim. C. Yes, because plaintiffs in federal court have the obligation to provide factual allegations supporting each element of their cause of action and to thereby establish a plausible claim. D. No, because the Federal Rules of Civil Procedure were promulgated to require only "notice pleading."

C. Yes, because plaintiffs in federal court have the obligation to provide factual allegations supporting each element of their cause of action and to thereby establish a plausible claim.

Casa Grande Homes, Inc., is a corporation that constructs expensive homes for the rich. It is incorporated and has its principal place of business in Oklahoma. After Casa Grande Homes builds a house, a related corporation known as Casa Grande Company, also incorporated and having its principal place of business in Oklahoma, seeks to sell the home to the highest bidder. Casa Grande Homes, Inc. constructed a $3 million home on outskirts of Dallas, Texas. In a contract entered between Casa Grande Company and Mike and Susan Pell, the Pells paid cash for this new house. After moving in, they discovered all sorts of defects in the construction, and neither of the Casa Grande entities would fix these problems. Shortly before the running of the applicable statute of limitations, the Pells, citizens of Texas, sued Casa Grande Company for the necessary repairs, which are estimated to be about $500,000. This suit was brought in the United States District Court for the Northern District of Texas and service was secured on the defendant within the limitations period. Twenty days after the running of statute of limitations, Casa Grande Company filed an answer in which it contended that the Pells' claim for defective construction could only have been brought against Casa Grande Homes, Inc., which built the home. In response to this defense, the Pells filed a motion for leave to amend their complaint to substitute Casa Grande Homes, Inc., as the defendant. They noted in their motion that they served an agent for service of process in Texas who represents both companies. The lawyer, who has long been employed by both companies, responded to this motion by contending that any amendment substituting Casa Grande Homes, Inc., would be futile. Should the federal court grant the Pells' motion for leave to amend? A. No, because plaintiffs cannot add new parties after the expiration of the applicable statute of limitations. B. No, because the amendment would be futile. C. Yes, because the amendment asserts the same claim against a new defendant who knew at the time of service that the Pells' were mistaken about the proper party's identity. D. Yes, because every motion to amend should be freely granted

C. Yes, because the amendment asserts the same claim against a new defendant who knew at the time of service that the Pells' were mistaken about the proper party's identity.

Casa Grande Homes, Inc., is a corporation that constructs expensive homes for the rich. It is incorporated and has its principal place of business in Oklahoma. The owners and executives of Casa Grande Homes (CGH) also own a related corporation known as Casa Grande Company (CGC), which is also incorporated and having its principal place of business in Oklahoma. CGC sells the homes constructed by CGH) to the highest bidder. CGH constructed a $3 million home on outskirts of Dallas, Texas. In a contract signed by Casa Grande Company and Mike and Susan Pell, the Pells bought this new house. \ After moving in, the Pells discovered all sorts of defects in the construction, and neither of the Casa Grande entities would fix these problems. Shortly before the running of the applicable statute of limitations, the Pells, citizens of Texas, sued Casa Grande Company in federal court in Texas for the necessary repairs, which are estimated to be about $500,000. The Pells filed the suit and served CGC within the limitations period. Twenty days after the running of statute of limitations, Casa Grande Company filed an answer in which it contended that the Pells' claim for defective construction could only have been brought against Casa Grande Homes, Inc., which built the home. In response to this defense, the Pells filed a motion for leave to amend their complaint to substitute Casa Grande Homes, Inc., as the defendant. They noted in their motion that they served an agent for service of process in Texas who represents both companies. The lawyer, who has long been employed by both companies, responded to this motion by contending that any amendment substituting Casa Grande Homes, Inc., would be futile. Should the federal court grant the Pells' motion for leave to amend? A. No, because plaintiffs cannot add new parties after the expiration of the applicable statute of limitations. B. No, because the amendment would be futile. C. Yes, because the amendment asserts the same claim against a new defendant who knew at the time of service that the Pells' were mistaken about the proper party's identity. D. Yes, because motions to amend should always be freely granted.

C. Yes, because the amendment asserts the same claim against a new defendant who knew at the time of service that the Pells' were mistaken about the proper party's identity.

Which of the following responses by the defendant to service of the complaint and summons constitutes a default? (There is more than one) A. Defendant does not answer, but files a motion to dismiss for improper venue within twenty-one days. The motion is denied. B. Defendant answers within twenty-one days and admits the allegations of the complaint C. Defendant's lawyer files an appearance within twenty-one days by submitting a paper to the clerk giving her name, address and bar number, and stating that she is appearing for the defendant D. Defendant herself brings a paper to the clerk saying that she plans to defend, and then shows up in court on the twenty first day saying she is ready to go to trial E. Defendant answers the complaint twenty-five days after service of the summons and complaint

D. Defendant herself brings a paper to the clerk saying that she plans to defend, and then shows up in court on the twenty first day saying she is ready to go to trial E. Defendant answers the complaint twenty-five days after service of the summons and complaint

George Planck was the railroad worker who was injured in the railroad accident described above. Although Mr. Planck was injured in the State of Washington, he brought his federal cause of action in a federal court against USNR in his home state of California. The railroad operates railroad lines in 28 States, and has 2,061 miles of railroad track in California (about 6% of its total track mileage of 32,500), employs some 2,100 workers there (less than 5% of its total work force of 43,000), generates less than 10% of its total revenue in the State, and maintains only one of its 24 facilities in California (4%). Mr. Planck argues that USNR clearly "does business" in California, which is the language of the California long-arm statute. Ms. Ahmad, as USNR's lead counsel filed a pre-answer motion to dismiss in the federal court in California. In this pre-answer motion, Ms. Ahmad contends that venue is not proper merely because the plaintiff resides in the district. She contends that this suit should be dismissed because of improper venue, but if the court disagrees she reserves USNR's right to file other grounds for dismissal. The California federal court denied her pre-answer motion to dismiss for improper venue, and Ms. Ahmad filed an answer in which asserted on USNR's behalf an objection (and a motion to dismiss) based on the California federal court's lack of personal jurisdiction. Should the federal court in California dismiss this suit for lack of personal jurisdiction? A. Yes, USNR was clearly not "at home" in California. B. Yes, Ms. Ahmad reserved her company's right to later object to personal jurisdiction. C. No, USNR is "at home" in California because it does a substantial amount of business in that state. D. No, USNR has waived its objection to personal jurisdiction.

D. No, USNR has waived its objection to personal jurisdiction

In the trial of a breach of contract action in federal court, the defendant moved for a motion for judgment as a matter of law after the plaintiff rested her case. He contended that the plaintiff had failed to provide any evidence to prove that she had performed her obligations under the contract, which is a required element of that cause of action. The judge denied the motion, and the defendant then presented his evidence. The case was given to the jury to decide, and it held in favor of the plaintiff and granted her $400,000 in damages for the defendant's breach of the contract. In a timely manner, the defendant filed what he entitled a "renewed motion for judgment as a matter of law" contending that the evidence overwhelmingly proved that his breach did not cause the damages alleged by the plaintiff. Proving causation was one of the elements of the plaintiff's cause of action. The plaintiff had provided evidence to support the conclusion that her business had suffered a $400,000 loss shortly after the defendant's breach of their contract. However, she had failed to offer any evidence proving that this loss was the result of the defendant's breach. The defendant had offered expert evidence to support the conclusion that the loss occurred because the pandemic-caused financial recession had undermined the plaintiff's business. Should the judge grant the defendant's post-verdict motion for judgment as a matter of law? A. Yes, a rational jury could not find for the plaintiff on this issue of causation if she failed to supply evidence to support that element. B. No, a jury could find for the plaintiff on this issue even if she had no evidence to support causation. C. Yes, the plaintiff cannot recover under these facts because she did not prove causation by a preponderance of the evidence. D. No, after the verdict, a defendant can only renew his earlier pre-verdict motion, but this was not a renewed motion because it was based on different grounds.

D. No, after the verdict, a defendant can only renew his earlier pre-verdict motion, but this was not a renewed motion because it was based on different grounds.

Pringle, while a citizen of Texas, bought a chain saw from Home Depot in San Antonio. While using the chain saw on trees at his home in San Antonio, the saw exploded causing him severe personal injuries. While recovering from his injuries, his wife was forced to take a new job that required both of them to move to New York. After his recovery, Pringle filed suit in a New York state court against Home Depot seeking $750,000 and asserting state claims for negligence and products liability. Home Depot is incorporated in Delaware and has its headquarters in New York, but nearly 20% of its revenue is derived from its stores in Texas. Home Depot filed a motion to dismiss this suit. Should the New York court dismiss this suit? A. Yes, because this cause of action arose in Texas and not in New York. B. Yes, because both parties are citizens of New York. C. No, because the New York state court has specific jurisdiction over this case. D. No, because the New York state court has general jurisdiction over this case.

D. No, because the New York state court has general jurisdiction over this case

Ten female employees of Toys for Kids, Inc. (Toys) have sued that company in a federal court in Texas asserting a federal cause of action for employment discrimination because of sex. The company is incorporated in Delaware and has its executive offices in Houston, Texas. Four of the plaintiffs are domiciled in states other than Texas, but six are domiciled in Texas. Each of the ten employees complains about a company policy applied in the different stores where they work that discriminates against women regarding promotions. Each contends that she was denied a promotion, which was then given to a less qualified male employee. Toys complains that each of the ten plaintiffs was denied a promotion at a different Toys' store, on a different date, and concerning promotion to a different job. It therefore moves to sever the claims into ten suits. Should the federal court grant this motion to sever these claims? A. No, because there are no restrictions on the free joinder of parties in federal court. B. Yes, because the litigation of ten claims in one suit poses an impossible burden on the court. C. Yes, because these ten claims arose from different actions by Toys taken at different times. D. No, because the plaintiffs complain about the same discriminatory company policy that creates a related set of transactions giving rise to a common issue.

D. No, because the plaintiffs complain about the same discriminatory company policy arising from a series of related transactions giving rise to a common issue.

Tesla Motor Company sued Lucid EV, Inc. in a federal court in California contending that Lucid infringed upon Tesla's patented configuration of lithium batteries that powers Tesla's cars. Lucid argued that Tesla's patent was invalid because of various defects in the application Tesla made to obtain the patent. After a full trial on the merits, the California federal court held that Tesla's patent on the configuration of lithium batteries was valid and had been infringed by Lucid. After this success, Tesla decided to file a second suit for infringement of the same patent in federal court in Michigan against Ford Motor Company. This suit is based on the contention that Ford has been infringing Tesla's patent on lithium car batteries. In answering this complaint, Ford asserted an affirmative defense contending that Tesla's patent was invalid because of defects in Tesla's application for the patent. Tesla responded to this defense by asking the Michigan court to strike Ford's challenge to the validity of Tesla's patent. Tesla contends that this issue has been fully adjudicated and that Ford should not be allowed to reopen the issue in this second suit. Is Ford precluded from litigating the patent's validity? A. No, issue preclusion always requires mutuality of parties. B. Yes, federal courts allow the offensive use of non-mutual issue preclusion when the same issue has been fully adjudicated in the first suit. C. Yes, Tesla has already litigated the issue of the patent's validity in federal court, and Ford is seeking to collaterally challenge that decision. D. No, due process prevents the use of issue preclusion against a party that has not had its day in court on the issue.

D. No, due process prevents the use of issue preclusion against a party that has not had its day in court on the issue.

General Motors, Inc. (GM), is incorporated in Delaware and has its principal place of business in Michigan. It sells its sports car, the Corvette, in Texas and nationwide. GM spends millions of dollars promoting this vehicle in Texas, which is one of the best markets for the Corvette. In Texas, it has hundreds of retail dealers who advertise, sell, and service both new and used Corvettes. Hillary Schrader was injured when the steering mechanism on her new Corvette caused the car to swerve into an oncoming car. The accident occurred in Houston in January of 2021. In May, the plaintiff, a citizen of Texas, brought a products liability state law claim in the United States District Court for the Southern District of Texas against GM seeking $2 million in damages. GM moved to dismiss this suit for lack of personal jurisdiction, contending that none of its purposeful contacts with Texas gave rise to the cause of action in Texas. GM has evidence to prove that Ms. Schrader purchased her Corvette when she was visiting her wealthy parents in Florida, who helped her buy the car. She returned to Houston with her new car and was trying it out at high speed on Interstate 10 outside that city when the steering malfunctioned. Should the federal court dismiss this suit? A. Yes, although GM has many purposeful contacts in Texas, its sale of the Corvette to her in Florida was the purposeful contact that caused the accident. B. No, GM is doing continuous and systematic business in the State of Texas and is therefore subject to the general jurisdiction of any court in the state. C. Yes, after GM's sale of the Corvette to Ms. Schrader in Florida it was her unilateral action that brought the car to Texas. D. No, the Texas court has specific jurisdiction because GM's many purposeful contacts with Texas are closely related to the cause of action asserted.

D. No, the Texas court has specific jurisdiction because GM's many purposeful contacts with Texas are closely related to the cause of action asserted.

Which one of the following cases can be removed from state to federal court? A. Paine, a citizen of Texas, sues Daniel, a citizen of Texas, on a state law claim for negligence, but Daniel (the defendant) asserted a federal cause of action as a counterclaim against Paine that arose from a common nucleus of operative fact. B. Paine, a citizen of Texas, sues Damon, a citizen of Utah, on a state law breach of contract claim in a Utah state court for $200,000. C. Paine, a citizen of Texas, sues Damon, a citizen of Utah, and Dolman, a citizen of Nevada, on a state law breach of contract claim in a Utah state court for $200,000. D. Paine, a citizen of Utah, sues Damon, a citizen of Utah, in a Utah state court on a cause of action created by a federal statute seeking $50,000 in damages.

D. Paine, a citizen of Utah, sues Damon, a citizen of Utah, in a Utah state court on a cause of action created by a federal statute seeking $50,000 in damages.

Which one of the following cases can be removed from state to federal court? A. Powers, a citizen of Texas, sues Daniel, a citizen of Texas, on a state law claim for negligence, but Daniel (the defendant) asserted a federal cause of action as a counterclaim against Powers that arose from a common nucleus of operative fact. B. Powers, a citizen of Texas, sues Damon, a citizen of Utah, on a state law breach of contract claim in a Utah state court for $200,000. C. Powers, a citizen of Texas, sues Damon, a citizen of Utah, and Dolman, a citizen of Nevada, on a state law breach of contract claim in a Utah state court for $200,000. D. Powers, a citizen of Texas, sues Damon, a citizen of Utah, on a claim authorized by a federal statute, in a Utah state court seeking $50,000 in damages.

D. Powers, a citizen of Texas, sues Damon, a citizen of Utah, on a claim authorized by a federal statute, in a Utah state court seeking $50,000 in damages.

Assume Sam Jones sued Susan Smith in a Texas court. Both own homes in San Antonio, and Mr. Jones knows Ms. Smith's address. Based on the Mullane decision, which one of the following methods of giving her notice would be in violation of due process. A. In-hand service of the summons and complaint on Ms. Smith by a sheriff. B. In-hand service of the summons and complaint on Ms. Smith by a private process server. C. Mailing the summons and complaint to Ms. Smith at her address by first-class mail. D. Publication of the summons and complaint in a San Antonio newspaper once each week for three weeks.

D. Publication of the summons and complaint in a San Antonio newspaper once each week for three weeks.

The Supreme Court in its ruling in United States v. Standard Oil found authority for federal common law to regulate the relationship between the United States Government and the members of the armed forces. How did it exercise that authority in Standard Oil? A. The Supreme Court held the Government could recover through a federal common law cause of action for indemnification. B. The Supreme Court held the President has the primary power over the military and therefore no federal common law power can exist. C. The Supreme Court held the California indemnification rule must apply because that is a substantive law under Erie. D. The Supreme Court held that the nature of the federal common law rule should be established by statute

D. The Supreme Court held that the nature of the federal common law rule should be established by statute

In July 2000, a week before the three year statute of limitations passes, Carson sues Herrera in federal court for breach of a contract to design a computer system for his store in Calpurnia, Illinois. In July 2001, he moves to amend his complaint to add a claim for violation of the state consumer protection act, based on the same dispute. The consumer protection act has a two year statute of limitations, which accrued at the same time as the three year statute. Which of the following is correct? A. The second claim would not be barred by the limitations period, as long as the judge grants the motion to amend B. The second claim would "relate back" to the date of the original filing of the case, and therefore would not be barred by the statute of limitations C. The second claim will be barred by the limitations period, because it will not "relate back" to the original filing under rule 15 D. The amendment will be barred, even if it relates back to the filing of the original complaint

D. The amendment will be barred, even if it relates back to the filing of the original complaint

Assume a federal statute includes a provision that creates a cause of action that can be asserted by an airlines company against any person who retaliates against it for refusing to grant cheap tickets. Also assume both parties have citizenship in the same state. The plaintiff in this suit, Mr. Gamez, filed his claim for breach of contract against United Airlines in a New York state court. Within 20 days after service in this state suit, United answered by asserting a counterclaim for damages alleging that Mr. Gamez 's suit constitutes "retaliation" under the new federal cause of action. In a timely fashion, United removed the whole case from New York state court to the United States District Court for the Southern District of New York. Mr. Gamez moved in federal court to remand the case back to New York state court. Should the federal court remand the case to state court? A. No, because Mr. Gamez 's claim satisfies the Grable exception as described in the Gunn decision and therefore can be removed. B. No, because Mr. Gamez 's claim is supplemental to United's federal question claim and therefore the whole case can be removed to federal court. C. Yes, because a federal court has discretion to remand any case removed to that court from state court when its docket is full. D. Yes, because a counterclaim cannot supply federal-question jurisdiction and no diversity exists.

D. Yes, because a counterclaim cannot supply federal-question jurisdiction and no diversity exists.

Patton, a citizen of Colorado, sued Dolby, a citizen of New Mexico. Patton has invoked diversity jurisdiction and seeks $200,000 in federal court in Colorado. Patton claims that Dolby, the broker in a real estate sale, fraudulently misrepresented the sewer system connection for a $1 million property in Colorado that he sold to Patton. Dolby filed a timely third-party complaint thereby impleading his principal in the sale, Tristan, a citizen of Colorado. Tristan was the original owner who sold the property to Patton. Dolby asserts that Tristan gave him incorrect information about the property's sewer system and therefore is liable to him for all or part of his liability to Patton. After Tristan was brought into the suit, Patton amended her complaint and asserted the same fraudulent inducement claim against Tristan, claiming that Tristan is liable to her for $200,000 in damages. Tristan responded by filing a motion to dismiss Patton's claim against him for lack of subject matter jurisdiction. Should the federal court dismiss Patton's claim against Tristan? A. No, because Patton could have joined Tristan in her original complaint as a defendant. B. No, because Patton's claim against Tristan arises from the same nucleus of operative fact as does her claim against Dolby. C. Yes, because supplemental jurisdiction can never support a claim asserted between nondiverse parties in a diversity case. D. Yes, because a plaintiff cannot use supplemental jurisdiction to circumvent the requirement of complete diversity in a diversity case.

D. Yes, because a plaintiff cannot use supplemental jurisdiction to circumvent the requirement of complete diversity in a diversity case.

Battery Inventors, Inc. (BII), has sued Tesla Motor Company contending that Tesla infringed its patent on lithium car batteries by using the same battery in the making of its fully electric cars in California. This suit was filed in a California federal court, which after a full trial on the merits of this federal statutory cause of action held that BII's patent was invalid. Battery Inventors decided not to appeal this decision because it did not want to establish a precedent. Instead, BII filed a second suit for infringement of the same patent in federal court in Minnesota against Hannington Batteries, Inc. (HBI). This suit is based on the contention that HBI has been infringing its patent on lithium car batteries by sales of its products in Minnesota. Hannington has asserted the affirmative defense of issue preclusion based on the California federal court's decision that BII's patent was invalid. Assume the general requirements for issue preclusion are satisfied, but BII contends that HBI cannot use the California court's decision reached in a case in which HBI did not participate. Hannington has moved for judgment based its claim that BII is precluded from re-litigating the issue of its patent's validity. Should the federal court in Minnesota rule that BII is precluded from re-litigating the issue of its patent's validity? A. No, issue preclusion requires mutuality of parties, and HBI was not in privity with Tesla. B. No, due process prevents the application of issue preclusion in favor of a party that did not participate in the first suit. C. Yes, the most significant factor is that the two suits involved the same claim. D. Yes, federal courts allow the defendant's use of non-mutual issue preclusion against a party that has had its day in court on that issue.

D. Yes, federal courts allow the defendant's use of non-mutual issue preclusion against a party that has had its day in court on that issue.

In 2020, Sara Layne, an attorney domiciled in Dallas, who is an expert in the law of business-entity dissolution, filed suit against Phil Duggan, a citizen of Oklahoma, in federal court in Texas. Ms. Layne seeks $150,000 because she contends that Mr. Duggan breached a contract the two parties had entered in Oklahoma. In 2020, Ms. Layne had phoned Mr. Duggan in Oklahoma and negotiated an agreement to dissolve his business partnership in Oklahoma. Ms. Layne and Mr. Duggan signed this contract in Oklahoma, and she has fully performed the contract in Oklahoma. However, Mr. Duggan has refused to pay her. Mr. Duggan has moved to dismiss this federal suit for lack of personal jurisdiction. At the outset of this suit, however, Ms. Layne had caused the federal court to seize a condominium in Dallas that Mr. Duggan owns and uses as a vacation home. Should this federal court in Texas dismiss the suit? A. No, Mr. Duggan's condominium serves as a basis for asserting general jurisdiction over him. B. No, Mr. Duggan's condominium serves as a purposeful contact that proves the court has specific jurisdiction over him. C. Yes, the totality of the contacts in this case were with Oklahoma and therefore no other state can assert jurisdiction. D. Yes, the ownership of the condominium is a contact in Texas, but it did not give rise to the cause of action asserted in this case.

D. Yes, the ownership of the condominium is a contact in Texas, but it did not give rise to the cause of action asserted in this case.

On February 2, 2017, the plaintiff and the defendant were involved in an automobile accident that occurred in Nevada. At that time, the plaintiff and the defendant were both domiciled in Nevada. In September 2018, however, the plaintiff moved to California to attend law school at the University of Southern California. Her husband and their two children moved with her, and they all live in an apartment in Los Angeles. The plaintiff's father is the senior partner in a large Las Vegas law firm, and she decided to enter law school because he offered her a job with his firm after graduation from law school. In anticipation of this job, she and her husband purchased a home in Las Vegas, and they plan to live in this home after her three-year stay in California. They currently lease this home, but that lease expires in three years. On the other hand, the plaintiff has often expressed her dream of being a clerk for a justice on the Supreme Court of the United States and of residing in Washington D.C. She has also stated that she would take a really good job in California if offered. She and her husband have both acquired California driver's licenses and have health insurance issued by a company in Los Angeles. After she started law school in 2018, the plaintiff filed suit against the Nevada defendant in a federal court in Nevada claiming he was negligent in causing her personal injuries in the February 2017 auto accident. All parties are citizens of the United States, and the plaintiff seeks $400,000 in damages. The Nevada defendant has moved to dismiss her suit for lack of subject matter jurisdiction. Should the federal court dismiss? A. No, the plaintiff is a citizen of California and the defendant is a citizen of Nevada at the time suit is filed. B. No, the federal court has supplemental jurisdiction over her claim against the defendant. C. Yes, the citizenship of the parties must be determined at the time of the accident, and they were both citizens of Nevada at that time. D. Yes, the plaintiff's move to California did not change her domicile because she moved there with the intent of returning to Nevada after a limited time.

D. Yes, the plaintiff's move to California did not change her domicile because she moved there with the intent of returning to Nevada after a limited time.

Imagine that you are a new mexico citizen and that while traveling to arizona, you enter a local hardware store looking for a chainsaw. You tell the owner that you intend to bring the chainsaw back to new mexico, and the owner then sells you the chainsaw. You return to new mexico, are injured by the chainsaw, and file a lawsuit in new mexico against the hardware store. Would it be constitutional for the new mexico court to exercise personal jurisdiction over the store? A. Yes, because the customer told the store owner where the chainsaw was being taken, the owner directed a contact to new mexico B. Yes, regardless of whether the owner knew where the chainsaw was being taken, the owner should have foreseen that a small product like a chainsaw could be used in a neighboring state C. No, although the claim arose out of the defendant's contact with the state, personal jurisdiction would be unfair and unreasonable in this case D. No, the claim does not arise out of the defendant's contacts with new mexico E. Yes, although the store's contact with new mexico is indirect, the reasonableness factors overcome the weak contacts that exist

E. Yes, although the store's contact with new mexico is indirect, the reasonableness factors overcome the weak contacts that exist


Ensembles d'études connexes

3.1 and 3.2 Civil Rights/Liberties

View Set

Professional Responsibility Final

View Set

ISM Week 3: Strategic Capabilities

View Set

vocabulary workshop level H unit 1-2

View Set

AP Stats: Experimental Design and Sampling Design

View Set